You are on page 1of 213

: IV

IV

, .
2011-14. .. .

: IV

1. .................................................................................................................................. 4
1.1. ............................................................................................................................. 4
1.2. .................................................................................................................... 7
1.3 ............................................................................................................ 13
1.4 ........................................................................................................................ 18
2. ............................................................................................................................................... 21
2.1. .......................................................................................................................... 22
2.2. ......................................................................................................................... 25
2.3. ......................................................................................................................... 30
2.4. (sin x)/x ............................................................................................................................. 33
2.5. ........................................................................................................................................... 35
2.6. ................................................................................................................................... 38
3. ................................................................................................................................................ 40
3.1. ........................................................................................................................... 40
3.2. .......................................................................................................................... 47
3.3. ............................................................................................................................... 52
3.4. ............................................................................................................................. 57
3.5*. ................................................................................................................... 60
3.6*. ................................................................................................................. 67
3.7*. ...................................................................................................................... 70
4. ............................................................................................................................................. 75
4.1. ................................................................................................................ 75
4.2*. ..................................................................................................... 82
4.3. .................................................................................................................... 87
4.4*. .......................................................................................... 96
4.5*. ..................................................................................................................... 99
4.6*. ................................................................................................................. 102
5. ................................................................................................................................ 115
5.1. ................................................................................................................................. 116
5.2. .............................................................................................................................. 119
5.3. ............................................................................................................................. 123

2

: IV
5.4. ........................................................................................................... 125
6. .................................................................................................................................... 135
6.1. ....................................................................................................................... 135
6.2. ............................................................................................................................. 139
6.3. .............................................................................................................. 144
7. ....................................................................................................................................... 153
7.1 .............................................................................................................. 153
7.2 ................................................................................................................. 161
7.3. ......................................................................................................... 171
7.4 ................................................................................................................................. 177
8. ........................................................................................................................................ 186
8.1. ........................................................................................................................................ 186
8.2. ............................................................................................................................ 191
8.3. ................................................................................................................................... 198
8.4. ................................................................................................................................. 206

: IV

1.
, ,
. , .
, ,

. , x y ,
z = xy.
1.1.
.
. ,
S = {(1,2), (1,3), (2,4), (3,4), (5,6)}
S : 12, 13, 24, 34 56.
A = { 1, 2, 3, 5},

B = {2, 3, 4, 5}.

, S. .
(f) (D) (K),
x D y K.
f : x y,

y = f(x).

(D) , ,
(K) , , .
, .
1.1.1. f : x y, x = y2,
, . , 9 3 ,
9, , 3, .
1.1.2. f : x y, x2 = y,
, . , 3 9 ,
.
f-1, ,
f, .
, A B,
B ,
, . ,

: IV
A B, 1-1 ,
.
B ()
, .
. , ,
, . .
()
y = x2 2x 3
.
f : x y
.

.
( x = 2)
( y = -3)
. ,

.
.

x = 0, y = -3. ,
. ,
x = 1,
- ()
( x > 1), .
, .
,
. ,
. :
(xD) f(-x) = f(x) ,
(xD) f(-x) = -f(x) .
, , y = x4 5x2 ,
y = x3 5x . . ()
, () .
( - ),
( ).
,
( ).

: IV

-, ,
. , , -.
, -, ,
.
f (a,b) D, x1, x2 (a,b)
x1 x2 f(x1) f(x2).
(<), .
f (a,b) D, x1, x2 (a,b)
x1 x2 f(x1) f(x2).
(<), (>),
.
, , . ()
x 1.58, (

-1.58, x = 0, x =

x < ) . () x -

-1.29, x (-

), x

1.29.

- () .
() , .
() ()
().

6

: IV
f m M (xD) m f(x) M. ,
(m)(xD) m f(x), f(x) M.
1.1.3.
1.

) .

1 0 1 2

f
3 2 3 1

) ?
) , ?
2. y = f(x)

2 x 1 3x 2
,
f

x 2 2x 3

f(2x 3) = 3x + 2,

1
2 f ( x) 3 f 2 x 3 .
x

, ?
3.
y = |x| - 1,
4.

y = |x 2| + 1,

y = |x + 1| - 2.

f(x) = x2 4x 5,
f(x) = x3 3x,
, ?

f(x) = x4 2x2.

5. ()

f ( x)

2x 1
,
3x 2

2 x 2x
f ( x)
,
2

f ( x) sin x cos x .

6.
y = sin x + cos x,

2 | x | 1
,
| x | 1

3x 2 12 x 10
.
x 2 4x 3

1.2.
.
. ,

: IV
: , , ,
,
1. (),
2. ,
3. .
, .
, .
, f(x) = 5 , .
- .

f(x) = anxn + an-1x n-1 + + a1x + a0,
an, an-1, , a1, a0 . ,
f(x) = x2 4x + 3,
. x = 1 x = 3.
, , (1 < x
< 3) . (2,-1),
. ,
.

f(x) = x3 3x2 x + 3.
x1 = -1, x2 = 1, x3 = 3,
( ) .
, = 0,
f(0) = 3,
, f .

,
()

3 2 3 0,15

3
2,15
( ) -
(),
(), +.
,
. ,
. , n n - 1
- n , . n .

: IV

, r(x) =

, g f

() . f g .
x = x0 , f(x0) = 0,
r(x). , , g(a) = 0, x = a
r(x). , x = a ,
, . , x = a
f(x) f(x) x .
1.2.1.

, :
1.

r1 ( x)

x2
x = -2 x = 1,
x 1

x2 4
2 x = 1,
x 1
x2
3. r3 ( x) 2
-2 x = 1.
x 1
2.

r2 ( x)

, , (-,0) ,
(0,+) .
() f = -2, x = 0.
, .
x0 r(x) =

, .

9

: IV
r(x) = f(x)/g(x) , f(x) g(x) .
1.2.2.

( x 1)( x 2)
x 1

.
( x 2)( x 2)
x2

, ,
, . x + 2 0. (x = 2)
, -1/2.
. ,
y(x) = x2 4x + 3 2. fm(x) gn(x) m n,
fm(x)/gn(x) m n.

. , .

.
1.2.3.

r ( x)

x 6 3x 4 x 3 5 x 2 4 x 7
.
x 4 5x 2 4

r ( x) x 2 2

x 3 x 2 4x 1
.
x 4 5x 2 4

A
Ax B
x 2
,
n
( x x0 )
( x px q) n

n = 1, 2, 3, , A, B, x0, p, q
p2 4q < 0, , .

10

: IV
, . g(x) = (x x1)(x x2)(x xn), r(x)

r ( x)

f m ( x)
An
A1
A2

...
g n ( x) x x1 x x2
x xn

A1, A2, , An . , .

g n ( x) ( x x1 ) j1 ( x x2 ) j2 ...( x xk ) jk , , j1 j2 ... j k n ,
r(x)

r ( x)

A1 j1
A11
A12

...

2
x x1 ( x x1 )
( x x1 ) j1

A2 j2
A21
A22

...

x x2 ( x x2 ) 2
( x x2 ) j2
...

Akj2
Ak1
Ak 2

...
.
2
x xk ( x xk )
( x x k ) jk

k,
r(x)

r ( x)

Ak x Bk
A1 x B1
A2 x B2

...
.
2
2
2
ax bx c (ax bx c)
(ax 2 bx c) k

.
f ( x) 5x 3 x , f ( x)

2x 1
, ... .
3x 2 2

1.2.4.
1. -2, 1, 3, 1.
2. 2 4, 4 ,
4.
3.

5
,
x4

2 x
,
3 x

x 2 2x
,
2 x

x2
.
x2 1


11

: IV
4. (2x4 + x3 - 3x2 + x 1) : (x2 2x).
[ 2x2 + 5x + 7, 15x 1, :

}
5. :

2 x 3 x 2 3x 1
,
f ( x)
x2 1

x3 x2 x
,
f ( x)
x2 x

x 4 x 3 2x 2 x
.
f ( x)
x2 x

6. :

2x 3
4 x 13
x2 4
,
,
y
(
x
)

,
y
(
x
)

x2 x
x2 x 6
3x 3 4 x 2 4 x
x 3 10 x 2 3x 36
x 2 29 x 5
x 4 5 x 3 6 x 2 18
,
,
y
y ( x)
(
x
)

.
y
(
x
)

( x 4) 2 ( x 2 3)
( x 1)( x 2 4) 2
x 3 3x 2
2
1
2x 1
5
x2
x
2 6
2
[,
,
, x2 2
.]
2

2
2
2
x x
x3
x 4 ( x 4)
x 3 x 1 x 4 ( x 4)
y ( x)

7*. a, b, p, q
x3 + 15x2 + 3x+ 5 = p(x a)3 + q(x b)3,

x3 + 15x2 + 3x + 5 = 0.
[x

2 3 3

2 3 3

8*. a, b, c

x 3 ax 2 bx c x 3 bx 2 cx a x 3 cx 2 ax b

x 1
x2
x3
.


12

: IV
[a = 4.2, b = -0.9, c = -4.3]
1.3
. :
1.
2.
3.
4.

,
,
,
.

1.3.1.
b > 0 , b 1. f : R R+
(xR) f(x) = bx
b.
,
(R)
. (R+)
.
, b > 1, f ,
b < 1, f .
, ()
e = 2,71828 ,

-1 0,36788.

.
, ,
.

1.3.2.
b
f(x) = logb x.
(R+) , .
, f(f-1(x)) = x f-1(f(x)) = x,
(x R+) b logb x x , (x R) log b b x x .


13

: IV
log b b 1 log b 1 0 . , (x1,x2 > 0):

log b ( x1 x2 ) log b x1 log b x2 , log b

x1
log b x1 log b x2 ,
x2

b x1 b x2 b x1 x2 b x1 : b x2 b x1 x2 ,
(x R) (a > 0) log b a x x log b a .
y
x

, log b a y , a b , . a b ,
x

y
log b a ,
x

. y x log b a . , ,
. , c > 0, 1

log b x

log c x
.
log c b

(log b x) (log c b) log c b logb x log c x .

log b c

1
.
log c b

, ,
,

I III (y = x).
:
e = 2,71828 -1 0,36788.
10,
,
(log naturalis), ,
:
log10 x = log x, ln x = loge x.

1.3.3.
, sin : R [-1,+1], 2.
, . ().


14

: IV
, cos : R [-1,+1], , 2.
, ,

- . , .
, tg : R R, . ,
. x =

, k Z.

, ctg : R R, .
, x = k, k Z.

-, .

y = sin x -

y = cos x -

y = tg x

y = ctg x

(xR):

sin 2 x cos 2 x 1 , sin x cos x , tg x ctg x .


2


15

: IV

sin( x y) sin x cos y cos x sin y , cos( x y) cos x cos y sin x sin y ,

tg( x y )

ctg x ctg y 1
tg x tg y
, ctg( x y )
.
1 tg x tg y
ctg y ctg x

sin 2 x 2 sin x cos x , cos 2 x cos 2 x sin 2 x , tg 2 x

2 tg x
.
1 tg 2 x

sin x sin y 2 sin

cos x cos y 2 cos

x y
x y
x y
x y
, sin x sin y 2 cos
,
cos
sin
2
2
2
2

x y
x y
x y
x y
, cos x cos y 2 sin
.
cos
sin
2
2
2
2

1
sin x sin y [cos( x y) cos( x y)] ,
2
1
cos x cos y [sin( x y) sin( x y)] ,
2
1
sin x cos y [sin( x y) sin( x y)] .
2
1.3.4.
.


16

: IV
,


,
,
2 2

. arc sin = sin-1 : [-1,+1]

sin(arc sin x) = x (|x| < 1), arc sin(sin x) = x (x [-/2, +/2].).


, [0,].
, arc cos = cos-1 : [-1, 1] [0, ].


, .
2 2

, arc tg = tg-1 : R

, arc ctg = ctg-1 : R (0, ).


,
/2 - .
1.3.5.
, , :

sh x

e x ex
e x ex
sh x
ch x
, ch x
, th x
, cth x
.
2
2
ch x
sh x

.
1.3.6.
1. -

3 x 3 x
,
2
tg x
,
f ( x)
x

f ( x)

3 x 3 x
,
2
cos x
,
f ( x)
x

f ( x)

f ( x) x 3 3x sin x ,
f ( x) 2 sin x 5 ctg x .

2. -

y ( x) log

3x 4
,
2x 3

y( x) log 2 (3x 2 5x 2) ,

y( x) 2 ln 2 x 7 ln x 4 .

3.

z ( x) 4 x 6 2 x 8 ,

z ( x) log

x 1
,
x 1

z ( x) [ln( x 3)]1 .

4.

y sin

2x
,
3

y tg x ctg 2 x ,

y sin

x
3x
cos .
2
5


17

: IV
5. f ( x) a sin x b cos x ,

3 sin x 4 cos x ,

(3 tg x 4 ctg x) sin 2 x .

2 cos x 5 sin x ,

6.

cos 1 x cos 1 ( x) ,

cos 1 x sin 1 x

tg 1 x ctg 1 x

1.4
y g, . y = y(g), g x, . g = g(x), y
x.
y = y[g(x)], y = f(x).
, f(x) = log2(x2 + 3x + 5) (g = x2 + 3x + 5)
(y = log2 g).
(. explicare , )

y f (x) .
, , .
. , ,
, , :

y 2x 3 , y x 2 2x 3 , y 9 x 2 .
(. Implicare , )

F ( x, y) 0 .
, .
,

x2 y2

1 0 .
25 16
,
. ,
, . , ,
-

xy cos y 0 .


18

: IV
, a y = f(x)
, . f

f ( f 1 ( x)) x , f 1 ( f ( x)) x ,
(f o f-1)(x) = (f-1 o f)(x) = x.
, (P) (a),
, , .

P a2 , a P .
y = x, I III
.
() y x 2 , 0 (), y

(), I III ( ). ()
y e x y ln x , = 2,71828... .

1.4.1.
1.

2 5 x 3 ,

5 tg x 3 ,

cos(5x 3) sin(5x 3) .

2.

xy 2 0 ,

b 2 x 2 a 2 y 2 a 2b 2 ,

2 x 2 3xy y 2 2 x 3 y 4 0 .


19

: IV
3.

m0

v
1
c

4. .
5.

y x,

x2 y2 r 2 ,

1
.
x

.
6.

x 2 3x 2 ,

x3 9 ,

x x.

x 2x 0 ,

sin x log 2 x .

7.

x 2 ( x 2) 2 8 ,


20

: IV

2.
x x1, x2, x3,
x0 |x x0|
, ,
, x0

lim xn x0 , xn x0 n .
n

x (a,b) x0
,
.
x1, x2, x3, x0 y1, y2, y3, y0. ,
lim xn x0 lim y n y 0 .
n

x n x0 n y n y 0 n ,
n 0 n 0 n . x1 y1 , x2 y 2 , x3 y3 , ... ,

xn y n = ( x0 n ) ( y0 n ) = x0 y0 n n , .
lim ( xn y n ) = x0 y0 0 0 = x0 y 0 .
n

lim ( xn y n ) lim xn lim y n .


n

,
. ,
.
: , , ,
, .
.
2.0.1. (xn) (yn) ,
1.

lim ( xn y n ) lim xn lim y n ,

2.

lim ( xn y n ) lim xn lim y n ,

3.

lim ( xn


: y ) lim x : lim y , lim y
n

0 .


21

: IV
2.1.
2.1.1.
f
c, , , c. L
f
x c, > 0 = () >
0, 0 < |x c| <
|f(x) L| < .

lim f ( x) L .
x c

1
. x
(c-, c+)
y (L-, L+).
, > 0
,
,
. f(x) L, x c, lim f ( x) L .
x c

(. 2.1.1.).
2.1.2.
f x0,

lim f ( x) f ( x0 ) .

x x0


2
. , lim ( x 4) = 32 4 = 5.
x 3

, f ( , ) x0,
, F
f .

lim f ( x0 ) = lim F ( x) F ( x0 ) .

x x0

x x0

2.1.3. (x R\{1})

: http://en.wikipedia.org/wiki/Limit_(mathematics).


22

: IV

f ( x)

x2 1
x3 1

= 1. , f(1) , =1
R . ,

x2 1
( x 1)( x 1)
x 1
= 2
.

3
2
x 1 ( x 1)( x x 1) x x 1
,
f , = 1 . ,
(xR)

F ( x)

x 1
.
x x 1
2

lim f ( x) lim F ( x)
x 1

x 1

2
.
3

f F, ( = 1)
.
2.1.4.

f ( x)

x 1 1
x 1 1

, x R\{0}.

= 0. ,
3

f(x) =

x 1 1
x 1 1

x 1

( x 1) 2 3 x 1 1

( x 1) 2 3 x 1 1

x 1 1
2

x 1 1
x 1 1

x 1 1

( x 1) 2 3 x 1 1

( x 1) 1
F ( x) ,
| x 1 | 1


23

: IV

x 1 1

F ( x)
3

( x 1) 2 3 x 1 1

, x R.

f ( x) F ( x) , x 0.
,

lim f ( x) lim F ( x)
x 0

x 0

2
,
3

, , = 0.
, ,

(: , , 0, - ).
2.1.5. lim

x 1 x .

x 1 x
x 1 x
. lim x 1 x = lim x 1 x
= lim
=

x
x 0
x 1 x
x 1 x x

( x 1) x
1
= lim
= 0, (),
lim
x
x
x 1 x
x 1 x
.
2.1.6.
1.

lim
x 5

lim x 3 3 .

3 3
,
x 5

x 0

2.

lim ( x 2 2 x 3) ,

lim

x 2

cos x
.
x

3.

x2 1
,
x 1 x 3 1
xm 1
lim n
, (m, n N)
x 1 x 1

x4 1
,
x 1 x 3 1
(1 x) n 1
lim
.
x 0
x

lim

lim

4.

24

: IV

lim ( x x 2 5 x ) ,

lim ( x 2 2 x x) ,

lim x x x x ,
x

lim ( x a)( x b) x .

5.
3

lim 4
x a

lim
x 4

x 3 a

x 4 a
8 x x2
84 x

lim 8

lim

x 6 x

x 12 x
x 2 25

5x 5

x 5

,
.

6.

lim 3
x 0

1 x 1 x
1 x 3 1 x
n

7. lim
x 0

lim

2 x 2 3 x 6

x 2

2 x 21 x

x 1 1
, n = 2, 3, 4, .
x
[ (a 1)(a n1 a n2 ... a 1) a n 1 a n x 1 ,
,
= lim
x 0

n 1

1
1
]
a ... a 1 n
n2

2.2.
.
2.2.1.
f x0,
.
: lim f ( x) A . (2.1.1.),
x x0

(A-, A+) , (x0-, x0+)


, x0,
, A < f(x) < A + .
2.2.2.
lim f ( x) A 0 , x0 ( x0) f(x) > 0.
x x0

: > 0, A > 0, A > 0. > 0


: x (x0-, x0+) f(x) (A-, A+). x0 f(x) > A > 0.


25

: IV
2.2.3.
lim f ( x) A lim g ( x) B lim [ f ( x) g ( x)] A B .
x x0

x x0

x x0

: > 0. |a + b| |a| + |b|,


|f(x) + g(x) (A + B)| = |[f(x) A] + [g(x) B]| |f(x) A| + |g(x) B|

(*)

lim f ( x) A 1 > 0 0 < |x x0| < 1 |f(x) A| < . ,


x x0

lim g ( x) B 2 > 0 0 < |x x0| < 2 |g(x) B| < .

x x0

min{1 , 2 } , 0 < |x x0| < |f(x) A| < |g(x) B| < ,


(*)

|f(x) + g(x) (A + B)| < + = ,


.

lim [ f ( x) g ( x)] A B .

x x0

2. , , f(x) g(x) = f(x) +


[-g(x)].
2.2.4.
lim f ( x) A lim g ( x) B lim [ f ( x) g ( x)] A B .
x x0

x x0

x x0

: >0. = 0 , A 0.

|f(x)g(x) AB| = |f(x)g(x) Ag(x) + Ag(x) AB| =


= |g(x)[f(x) A] + A[g(x) B]|
|g(x)||f(x) A| + |A||g(x) B|. (*)
lim g ( x) B 1 > 0 0 < |x x0| < 1 | g ( x) B |
x x0

2| A|

(x0-1, x0+1) ( 2.2.1.); |g(x)| < M, M > 0.


, lim f ( x) A 2 > 0 0 < |x x0| < 2 | f ( x) A |
x x0

2M

= min{1, 2}
2

, . wiki.elemenat.com, , : .


26

: IV

| g ( x) B |

2| A|

, |g(x)| < M | f ( x) A |

2M

(*) 0 < |x x0| <

| f ( x) g ( x) A B | < M

2M

| A|

2| A|

.
2.2.5.
lim f ( x) A , lim g ( x) B B 0 , lim [ f ( x) : g ( x)] A : B .
x x0

x x0

x x0

: (3 4.)

| f ( x ) : g ( x) A : B | =

B f ( x) A g ( x)
f ( x) A
=
=

B g ( x)
g ( x) B

1
B [ f ( x) A] A [ g ( x) B]
| B | | g ( x) |

1
| A|
| f ( x) A |
| g ( x) B | .
| g ( x) |
| B | | g ( x) |

.
, (, , ) (,
, ) .
, . ,
, , .
2.2.6.
lim f ( x) A , lim g ( x) B x0, x0,
x x0

x x0

f(x) g(x), A B.
: , A > B. 2.2.1.

lim [ f ( x) g ( x)] = lim f ( x) lim g ( x) = A B 0 ,

x x0

x x0

x x0

2.2.2. x0 f(x) g(x) > 0, . f(x) > g(x). ,


f(x) g(x).


27

: IV
, . , A > B,
, .
2.2.7.
lim f ( x) lim h( x) A x0, x0,
x x0

x x0

f(x) g(x) h(x), lim g ( x) A .


x x0

: f(x) A g(x) A h(x) A -


x0, 0 < |x x0| < . , |g(x) A| max{|f(x) A|, |h(x) A|} ,
. > 0.
, 1, 2 > 0
0 < |x x0| < 1 |f(x) A| < ,
0 < |x x0| < 2 |h(x) A| < .
* = min{, 1, 2},
0 < |x x0| < * |g(x) A| < ,
. lim g ( x) A , .
x x0

.
2.2.8.
1.
2.
3.
4.
5.

lim C C , C .

x x0

lim Cf ( x) C lim f ( x) .

x x0

lim f ( x)

x x0

x x0

g ( x)

lim f ( x) x x0

lim g ( x )

x x0

lim n f ( x) n lim f ( x) .

x x0

x x0

lim log b f ( x) log b lim f ( x) .


x x0

x x0

3 2.2.9.

x 1 x2 1
.
lim 2
3
x 1 x 1
x 1

, : , IV
, , 1994.


28

: IV
:

lim
x 1

lim
x 1

x 1
x 1
1
1
= lim
= lim

2
x

1
x

1
x 1 2
( x 1)( x 1)
x 1

x2 1
( x 1)( x 1)
x 1
2
= lim
= lim 2

3
2
x

1
x

1
x x 1 3
x 1
( x 1)( x x 1)

2.2.3.

x 1 x2 1 1 2 7
= .
lim 2
3
x 1 x 1
x 1 2 3 6

3
1
3
.
x 1 x 1
x 1

2.2.10. lim

: 2.2.3. .
, ,

x2 2
( x 1)( x 2)
1
3
=
=
.
3
x 1 x 1 ( x 1)( x 2 x 1) x 2 x 1
, 1 1.
3

x 1 1

x 0 4

x 1 1

2.2.11. lim

: ( 2.1.6.) 7.
3

= lim

x 0 4

3
x 1 1
x 1 1
lim
1
x 0
4
x
x
=
= 3 .
4
3
x 1 1
x 1 1 1
4
lim
x 0
x
x

2.2.12.

1. ( ):

x8 2x 6 3
.
x
3x 8 2

3x 2 4 x 5
,
x 5 x 2 6 x 7

lim

lim

2. ( ):


29

: IV

3x 2 13x 10
,
x 5 2 x 2 13 x 15

2 x 2 13x 7
.
x 7 3 x 2 19 x 14

lim

lim

3. :

1
24
2
lim
2
3
,
x 2 x 2
x 2x 4 x 8

3x 2 x
2
.
lim

x 2 ( x 2)( x 2 x 1)
x 2

, ]

4. ( ):

lim x 2 x 1 x 2 x 1 ,
x

lim 3 ( x 1) 2 3 ( x 1) 2 .
x

5.

lim
x

1 sin 2 x
,
1 cos 4 x

lim
x

tg x
,
sin 2 x

sin x
lim
tg 2
cos 2 x
x
2

1 tg x 1 tg x
lim
,
x
sin 2 x

x .

6.

lim a 2 a1 x b1 x 2 a2 x b2 ... x 2 an x bn nx ,
x

lim x x x x x .
x

2.3.
2.2.1.
L f x0, > 0
= () x0 < x < x0 |f(x) L| < .
lim f ( x) f(x0 0).
x x0 0

(2.2.1.) x0 < x < x0 x0 < x < x0 , L


. lim f ( x) f(x0 + 0).
x x0 0


. , ,
. ,
( L), ( L).


30

: IV
2.2.2.
f(x) = [x], . ,
f(-2,03) = [-2,03] = -2, f(-1,98) = -1, f(1,99) = 1, f(3,01) = 3.
, f : x [x] = n (n x < n+1, n Z).
, x0 = 1
. , f(1 0) = 0, a f(1 + 0) = 1. , x0 = 1
, . f(1) = 1.
0, ,
.
f(x) r, . (0 < |x x0| < r,
0 ( 0) , f(x) > 0,
,

lim

x x0

1
0,
f ( x)

0 +

lim f ( x) .

x x0

, f(x) r, 0
( 0) , f(x) < 0,
, 0 -

lim f ( x) .

x x0

.
2.2.3.
f 0, 0. > 0
() > 0
0 < |x x0| < () f(x) > ,
f + 0 lim f ( x) .
x x0

lim f ( x) .
x x0

2.2.4.

31

: IV

1
1
1 x
1
0 = 0 lim , lim , f ( x)

x 0 x
x 0 x
1 x
x
1 x
1 x
0 = 1 lim
, lim
.
x 10 1 x
x 1 0 1 x
f ( x)

, . ,
[r, +), r, lim f ( x) L
x

1
lim f L , , .
t 0
t
2.2.5.
f , [r, +). > 0
() (),
x > () |f(x) L| < ,
L +

lim f ( x) L .

-.
2.2.6.
1.

x 2 1, x 0
f ( x)
2 x , x 0
0 = 2.
x

1
2. lim 0 .
x 3

5 3x 2
.
x x 2 1

3. lim

a m x m a m1 x m1 ... a1 x a0
.
n 1
x b x n b
... b1 x b0
n
n 1 x

4. a b lim

5. lim

1 x .


32

: IV

1
.
x x0 ( x x ) 2
0

6. lim

2.4. (sin x)/x


2.4.1. ,

lim
x 0

sin x
1.
x

: , ( , 1)
x = BOD, , B OD .
OB AD BC, A C
( OD).
, OD = 1, AD = sin x. OB = 1, BC = tg x.
, OBD, O , OBC

P(OBD)

1 AD sin x
x
tg x
x
, P(O BD )
.
(r 2 ) , P(OBC )

2
2
2
2
2

P(OBD) P(O BD) P(OBC ) ,


sin x x tg x
,

2
2
2
| sin x || x || tg x | .
|sin x|,

|x|
1
, .

| sin x | | cos x |

| sin x |
| cos x | .
| x|

, (0 < x < ) .

lim 1 1 lim cos x 1 , 2.2.7 (. Squeeze theorem), lim


x 0

x 0

x 0

sin x
1 ,
x

.
f(x) x0 f(x0) = 0, x x0


33

: IV

sin f ( x)
1.
x x0
f ( x)
lim

2.4.2. :

sin x 2
x0
x2

sin 3x
x 0
5x

1. lim

2. lim

: 1. f ( x)

sin 3x
,
5x

, 3/5. f(0,1) = 0,591..., f(0,01) =


0,59991..., f(0,001) = 0,5999991..., f(x) 0,6 x 0.
, : lim
x 0

2. f ( x)

sin 3x 3
sin t 3
sin 3x 3
= lim
= lim
.
t

0
0
5
5
t
5x
3x
5

sin x 2
( ) ,
x2

1. : f(0,1) = 0.999983, f(0,01) = 0.9999999983 .

sin x 2
sin t
, t = x 0 ( x 0), lim
= lim
1 .
2
x0
t

0
t
x
2

2.4.3.
1. :

sin x
3

lim
,

x
x

3
3

sin(x)
,
x 0
x

sin(sin x)
.
x 0
x

lim

lim

[ arg(x) = t 0]
2. :

1 cos x
,
x 0
x2

tg x
,
x 0
x

lim

sin(tg(sin x))
.
x 0
x

lim

lim

3. :

sin x
lim
x

2
2 ,

lim
x

2 cos x 1
.
1 tg 2 x

4
[: x = /4 + h, h 0, ]

34

: IV
4. :

lim
x 0

arcsin 3x
,
2x

lim
x 0

arctg 2 x
.
3x

5. :

lim
x 0

lim sin x 1 sin x .

1 sin x 1 sin x
,
x

x 0

6. :

lim x arcsin
x
2

x2 1
x

lim arcsin
x

1 x
.
1 x

2.5.
x

1
2.5.1. lim 1 e .
x
x

= 2,71828... , ,
.

: +. = lim 1

1
, n N.
n

x 1, [x] ,
[x] x < [x] + 1.

1
1
1
,

[ x] 1 x [ x]
1

1
1
1
,
1 1
[ x] 1
x
[ x]

1
1

[ x] 1

[ x]

1
1

1 1
x

[ x]
x

[ x ]1

lim 1
[ x ]
[ x] 1

[ x]

= lim 1
[ x ]
[ x] 1

[ x ]1

1
[ x] 1


35

: IV

1
= lim 1

[ x ]
[ x]

[ x ]1

1
= lim 1

[ x ]
[ x]

[ x]

1
= e 1 e ,
1
[ x]

1
2.2.7. lim 1 e .
x
x

x -, y = -x +,

1
1
lim 1 = lim 1
y
x
y
x

y
1

= lim 1
= lim
y
y y 1
y 1

y 1

y 1

= lim
y
y

1
= e 1 e .
1
y 1

1
y 1 () , = 6,
x

2,5 3 (, = 2,71828...).
2.5.2.

lim 1 x x ;
1

1.

x 0

ln(1 x)
;
x 0
x

2. lim

3.

ex 1
.
x 0
x

lim


36

: IV
1
1
ln(1 x)
1
x
lim 1 x x = lim 1 = ; 2. lim
= lim ln 1 x x = ln e 1 ; 3. t = e 1, x =
x 0
x

0
x

0
t
x
t
x
1
e 1
t
1
ln(1 + t), t 0, lim
= lim
= lim
= 1 .
t

0
x 0
t

0
ln(
1

t
)
x
1
ln(1 t )
t
t

1.

2x 3
2.5.3. lim
.
x 2 x 5

3x

2x 3
: lim

x 2 x 5

3x

(2 x 5) 2
= lim
x
2 x 5

2 x 5 2

2 2 x 5
2

lim 1

2
x

3x

3x

= lim 1

x
2x 5

2 x 5

2
2

= lim 1

2
x

3x

6x
2 x 5

lim e

6x
2 x 5

lim e

6
5
x

-3

= .

2.5.4.
1.

x 1
lim
,
x x 1

2x 1
lim

x 2 x 3

2x

x2

2.

(1 x) 1
.
x 0
x

ln(1 x )
x
[ t a 1 ; (1 x) e
]

a x 1
,
x 0
x

lim

lim

3.

ex e x
,
lim
x 0
x

1
x2

lim (cos x) .
x 0

4.
x

x2 x 1
,
lim 2
x x x 1

x 2 3x 1
.
lim 2
x x 3 x 1

ex 1
,
x 0 sin x

lim x( x a 1) , a > 0.

5.

lim

6.


37

: IV

lim
x 0

sin 2 x
1 x sin x cos x

lim

sin x
2
,
lim

x
x
2
2 2 cos
2

2 cos x 3
,

sin x
6

lim (1 tg 2 x ) 2 x .
x 0

2.6.
(. ) .
, .
f : D R

x = x0 lim f ( x) , ;

y = n lim f ( x) n ;

y = kx + n, k 0, lim[ f ( x) y] 0 ,

x x0 0
x

k lim

f ( x)
, n lim[ f ( x) kx] .
x
x

2.6.1. f ( x)

x2
.
1 x

. y = n, n lim f ( x) . ,
x

, , .
, .


38

: IV
x = 1
,
f(x) + x 1-0,
f(x) - x 1+0.
()
(),
x = 1.
,
.
y = -x 1,

x2
1 ,
x 1x ( x)

k lim

x2

n lim
x 1 .
x 1 x

()
, -
+.

2.6.2. .

1
x
x3
4. f ( x)
x4
x2
7. f ( x)
x 1
x
10. f ( x) 2
x 4

2. f ( x)

13. f ( x)

14. f ( x) arctg x

1. f ( x)

x2 1

2
x3
2x 1
5. f ( x)
2x 1
x2 1
8. f ( x)
2x 1
4
11. f ( x) 2
x 1

3
2x 1
2x 3
6. f ( x)
3x 4
x2 1
9. f ( x)
2x 1
x2 1
12. f ( x) 2
3x 2
2x 2 2x 5
15. f ( x) 2
.
x 3x 4
3. f ( x)


39

: IV

3.
y = f(x) x1 x2 x = x2 x1
, y = f(x1+x) f(x1) .

(1, 2).

T1(x1, y1) T2(x2, y2). , x 0


- (1 = 2).
, x 0,
y =

. (16421727) , (1646-1716) .
3.1.

, .
.
y = f(x)

f ' ( x) lim

x 0

f ( x x) f ( x)
,
x

.
, -
. y = f(x)
x = x2 x1 y = f(x2) f(x1),
ABC.
A

tg A

f ( x2 ) f ( x1 ) y
BC

=
.
x2 x1
x
AC

- t, .
A B.
x 0 A B
t
x2 = x1.
3.1.1. f(x) = C,

C 0 ,
. .
: f(x+x) = C, f(x+x) f(x) = 0, f ( x) lim

x 0

f ( x x) f ( x)
= 0.
x


40

: IV
, - C,
. .
3.1.2. f(x) , Cf(x), C ,

Cf ( x) Cf ( x) .

f ( x x) f ( x)
Cf ( x x) Cf ( x)
= C lim
= Cf (x) .
x 0
x 0
x
x

: Cf ( x) lim

3.1.3. u(x) v(x) ,

u( x) v( x) u ( x) v( x) .
f(x) = u(x) + v(x) ,

u ( x x) u ( x) v( x x) v( x)

= u ( x) v( x) .
x
x

f ( x) lim
x 0

u ( x) v( x) u ( x) v( x) .

(u v) u v .
. , (u v + w) = u v + w.
3.1.4. u(x) v(x) ,

u( x)v( x) u ( x)v( x) u( x)v( x) .


: f(x) = u(x)v(x),

f ( x x) f ( x) = u( x x)v( x x) u( x)v( x) =

u( x x)v( x x) u( x)v( x x) u( x)v( x x) u( x)v( x) =


[u( x x) u( x)]v( x x) u( x)[v( x x) v( x)] .

[u( x)v( x)] =

v( x x) v( x)
u ( x x) u ( x)
lim
v( x x) u ( x)
=
x 0
x
x

u( x)v( x) u( x)v( x) .

41

: IV
, (uvw) = uvw + uvw + uvw.
, .

, F(x,y) = 0. , xy = 1
, x = 1, y + xy = 0, y
y
y

y y0

y
.
x

1
,
x

1
. T(x0,y0)
x2

y0
( x x0 ) , . x0 y xy 0 2 x0 y0 ,
x0

x0 y xy0 2 .
3.1.5. v(x) ,

v(x) 0:

v
1
2 .
v
v
: v(x) , v(x) 0
. f ( x)

1
.
v( x)

+ , v(x+x) 0.

f ( x x) f ( x) =

1
1
v( x) v( x x)
=
,

v( x x) v( x)
v( x x)v( x)

1
v( x x) v( x)
f ( x x) f ( x)
=
.

x
v( x x)v( x)
x
, lim v( x x) v( x) , .
x 0

3.1.6. u(x) v(x) ,

v(x) 0:

u u v uv
.

v2
v
:

42

: IV

1
1
u uv u v uv
u 1

.
= u = u u = 2 =
v v
v
v2
v v
v
( 4. 5.).
3.1.7. x ( y) , y f (x) , f (x)

f x( x) 0 , (x) y ( y ) , :
f x( x) y ( y) 1 .
: f f (x) (, ), .
, x ( y) . ,
0, 0, . ,

f ( x x) f ( x) y , ( y y) ( y) x ,

f ( x x) f ( x) y ( y y) ( y) x
,
,

y
y
x
x
.

f ( x x) f ( x) ( y y) ( y)

1.
x
y
f (x) f x ( x) 0 ,
( y ) , y ( y ) , .
, y y(x) x x( y)

dy dx
x dx
y dy
x y lim
,

1.

x 0 x
y 0 y
dx
dx dy
dy

y x lim

, y x 2 , x 0, x

y x 2 x x y

y x x y 2 x

1
2 y

y , y 0,
1

2 y

, .

x
x2

1,


43

: IV
y x 2 ,
-.
, y y( x) e x ( y) ln y () . ( ,
) y x e x , y ( y )

1
,
y

y x y ( y) e x

1
1
y 1.
y
y

e x y .
3.1.8. y f (g ) f g (g ) g, g g (x) g x (x) ,
y f [ g ( x)] :

f [ g ( x)]

f g ( g ) g x ( x) .

: y f (g ) g g (x) , y f [ g ( x)] F ( x) .

f (g ) f g (g ) g, g (x) g x (x) x. f (g ) g (x)


g x. x x + x, g g g ( x x) g ( x) ,
g (x) , g 0 x 0 .

F ( x x) F ( x) = f ( g g ) f ( g ) ,

f ( g g ) f ( g ) g ( x x) g ( x)
F ( x x) F ( x)
=
,

x
g
x

lim

x 0

f ( g g ) f ( g )
g ( x x) g ( x)
F ( x x) F ( x)
= lim
.
lim

0
x
g
x

, . ,

y f (v) , v v(u) , u u(x) ,


y f v(v) vu (u) u x ( x) .
, y ( x 2 x) y ( x 2 x) 1 ( x 2 x) = (2 x 1)( x 2 x) 1 .


44

: IV

, y ln g ( x) y

1
g ( x) .
g ( x)

, y tg(sin x) , y

cos x
.
sin 2 x

3.1.9. y = x3 , , x0
y = y(x0+x) y(x0) = (x0+x)3 (x0)3
= 3x02x + 3x0(x)2 + (x)3
= 3x02x + (3x0x + (x)2)x
= x + (x)x,
= 3x02 x, (x) = 3x0x + (x)2 0 x 0.
, ,

y
(x) , x 0,
x
x.

o
.

A(x, f(x)), B(x+x, f(x)+dy),
C(x+x, f(x)), tg = f(x).
y = x3 x
dy = 3x2x.
3.1.10. y = f(x) x0
y, x ,
y = x + (x)x,
x, lim (x) 0 . x
x 0

f x0 dy = x.
,
. , f(x+x) f(x) f(x)x
f(x+x) f(x) +f(x)x.
, .
, .


45

: IV
3.1.11. , y = f(x) x0 ,
, :

y
y
, f lim
.
x 0 x
x 0 x

f lim

x0
(, ).
3.1.12. f(x) = |x| 1 .
:

x 1, x 0
f ( x)
x 1, x 0,

y (| 0 x | 1) (| 0 | 1)
=
=
x
x
| x | 1, x 0
=
x
1, x 0.
f 1 f 1 . , ,
x = 0 -1,
1, .
3.1.13.
1. , .
2. 3.1.2 3.1.1 4.
3. 3.1.5. 3.1.6. 1.
4. x ( y) y f (x) , f x (x)
-, y ( y ) -,
Oxy . , + = /2, + =
/2 + . tg = ctg , . f x( x) y ( y) 1.
5.

4,03 .
x 1, x 1

6. f ( x)

x 1,

x 1

?

46

: IV
7. y 3| x| .
3.2.
(1 10.) (1118.) .
3.2.1.
1. C 0 , C .
4.

2. x 1 .

x 2 1 x .

5. x

8. log b x

7. e x

1
x

1
.
x2
1

13. tg x
.
cos 2 x
1

16. arccos x
.
1 x2
10.

2 x .
a a ln a .

3. x 2
1

, R.

1
.
x ln b

11. sin x cos x .

14. ctg x

17. arctg x

1
.
sin 2 x
1
.
1 x2

6.

9. ln x

1
.
x

12. cos x sin x .

15. arcsin x

1 x2
1

18. arcctg x
.
1 x2

:
4.

x = lim

lim

x 0

x 0

( x x) x
x x x
x x x
x x x
= lim
= lim
=

x 0
x
x
x x x x0 x( x x x )

1
x x x

1
.
2x

7. f ( x) e x

e x 1
f ( x x) f ( x) e x x e x
=
= ex
,
x
x
x

e x 1
1 (. 2.5.2. 3.), e x e x .
x 0
x

lim

9. ln x 7. ( f ( x) e x ) . 3.1.7. ln x x e y

ln x x

e
y

=
y

1,

1 1
.
ey x


47

: IV

lim ( x x)x

5. = 1, 2, 3, x

x 0

( x x x)( x x) 1 ( x x) 2 x ( x x) 3 x 2 ... x 1
=
x 0
x
lim ( x x) 1 ( x x) 2 x ( x x) 3 x 2 ... x 1 = x 1 .
lim

x 0

, , , x > 0, x e ln x ,
7. . ln x u , x e u .

= e ln x

1
= x 1 , .
x

x x < 0 ( , ), x (1) ( x) ,

. u = -x, 3.1.2. 3.1.8. x (1) u , x x =


3.1.8. x

= x

(1) u 1 u = (1) ( x) 1 = (1) 1 ( x) 1 = x 1 . , x

x 0 : x x 1 , > 0, = 0 ,

: ( x) 1 , x

2x ,

1
1
, 2 , .
x
x
2 x x

6. a x e x a x e x ln a .

= e

, u ln a , a x

a x 1 ln a , . a x
8. log b x

u x = e u x ln a x = e x ln a x x ln a =

ln a .

ln x
1

log b x
.
x ln b
ln b

11. f(x) = sin x f(x+x) f(x) = sin(x+x) sin x = 2 cos x

x
x
,
sin
2
2

x
sin
x 2
f ( x x) f ( x)

= cos x
. , 2.4.1.

2 x
x

(sin x / x 1, kada x 0), sin x cos x .

x cos x cos x (1) sin x .


2

12. cos x sin


48

: IV

(sin x) cos x (cos x) sin x cos 2 x sin 2 x


1
sin x
13. tg x =
=
=
, .
=
2
2
cos 2 x
cos x
cos x
cos x
1

tg x
.
cos 2 x

14. , ctg x

cos x
.
sin x

15. y arcsin x x sin y . arcsin x x sin y

arcsin x x cos y 1 , arcsin x =

1
=
cos y

1 sin 2 y

1
1 x2

1 , .

cos y.
16-18.
.
3.2.2.
1. , x ?
x = 0,

x.

2. e x ? ?
3. y ln x ? ?
4. ( ):
. 3x 2 2 x 3 ,

. 5 2 x 3x 4 ,

. ax 5 bx ,

. 3 mx 2 2nx 3 ,

. (bx 2 c) m ,

. 4 x13 x15 ,
. ( x a) n

1
,
x

1
.
(x ) p

5. :
3
. 1 5 x 23 x 4 x ,

. xe ,

. x 10 ,

. ln ln x ,

. log b f ( x) ,

. ln x

x2 1 ,

. 2e x 3a x ,

. (3 x 3 x ) 4 ,

. log

x x
. 2 x 3 x ,

ax
.
,
x
. log(1 x 2 ) ,

x x

. a3 x 2 b4 x ,

. log b (kx n) ,
. xa x 3x 2 e x .


49

: IV

6. 1 2 x 3x 2 ...nx n1 =

nx n 1 (n 1) x n 1
.
( x 1) 2
n

k 1

x n 1 1
]
x 1

7. f ( x) f ( x0 ) f ( x0 )( x x0 ) x0 = 0 (1 x) ,

e x ln(1 x) .
8. y sin x - : 30, 45, 60.
-? cos x, tg x, ctg x.
9. : . , . -?
10. :
. sin 3x ,

. tg( x ) ,

. sin 3 x ,

. x cos x ,

. e x sin x ,

. m sin ax n cos bx ,

x
3

. ln tg ,
11. sec x

x
,
2 3

. log b cos

. cos 2 x ,

sin x
,
x
. log sin x ,
.

. ex (sin ax b) .

1
1
csc x
.
cos x
sin x

12. f ( x) f ( x0 ) f ( x0 )( x x0 ) x0 = 0 ,
.
13. (0, -6) (0,
6).
14. :
. arcsin

x
3

x
,
3

. arctg ,,

. arctgx ln x 2 1 .

15. , .
16. ,

y xx ,

y x sin x ,

y (sin x) cos x .

17. y = f(x) :

x 2 5xy 3 y 2 5 0 .

50

: IV
[ F(x,y) ( )
, '.
2 x 5 y (5x 6 y) y 0 , y

2x 5 y
]
5x 6 y

x2 y2
18. 2 2 1 .
a
b
[ ,

b2 x
2x 2 y

,
,
.

0
a2 y
a2 b2

T(x0, y0) y y 0

b2 x
( x x0 ) ,
a2 y

a 2 yy 0 b 2 xx 0 b 2 x02 a 2 y02 ,
a 2 yy 0 b 2 xx0 a 2 b 2 , .

xx 0 yy 0
2 1]
a2
b

19. ' , , :

x2 y2

1,
a2 b2

x2 y2 r 2 0 ,
2
3

2
3

2
3

y 2 2 px ,
x 3 y 3 3axy 0 .

x y a ,
.

20. ( x 2 y 2 ) 2 a 2 ( x 2 y 2 ) -.
21. x a cos t , y b sin t .
[ x f1 (t ) , y f 2 (t )

f (t t ) f 2 (t ) f1 (t t ) f1 (t )
y f 2 (t t ) f 2 (t )

:
= 2
.
x
f1 (t t ) f1 (t )
t
t
f1 f2 x f1(t ) , y f 2 (t ) ,

y y
, x 0 .
x x
, , x f1 (t ) , y f 2 (t ) x y t,
lim

x 0

y
, x 0 .
x
y
b cos t
b
, y =
= ctg t .]
x
a sin t
a
y x


51

: IV
22. (. 21.).
[ y y0 y ( x0 )( x x0 ) ] , ' .]
23.

x a(t sin t )
,

y a(1 cos t )

x a cos 3 t
.

3
y

a
sin
t

3.3.
, , .
. .
. ,
x-. y = f(x), f(x) = 0.
, = 0 F(x,y) = 0.
, y 3 x 2 1 (1, 0), x-.
y < 0, y > 0, .
, .
, y 3 x 2 1 -1 < x < 1, x (-, +).
. . ,
.
y f (x) ,
: x1 , x2 - x3 .

, .
-.
,

.
, .
. y 0 , -,
, , .
, x (-, x1) (x3, +). y 0 ,
. x (x1, x3)\{x2}.


52

: IV

,

,
. ()
--, --.
,

y 3 x 2 1 , y

2x
3 3 ( x 2 1) 2

.
()
x = 0 .
= 1 ,
. (
),
= 1.
, y 3 x 2
R, ,
, y

1
3 3 x

= 0. , = 0,
( ),
.
, .
, (0, 0) .

.
,
= 0 .
.
.
, .
: , , . , ,
. ,
.
, . -. , , .


53

: IV
, , . ,
--, --.
, , , -,
, .
, .
3.3.1. f ( x) 2 x 3 3x 2 12 x .
: , f ( x) x(2 x 2 3x 12) ,
x = 0,

3 105
1,81 -3,31. -.
4
, , ,

f(x) < 0
- < x < -3,31 0 < x < 1,81 ().
-.
f(x) > 0 -3,8 < x < 0 1,81 < x < +.
-.
f ( x) 6 x 2 6 x 12 = 6( x 2)( x 1) .
x1 = -2 x2 = 1
.
y1 f ( x1 ) = f (2) , .
y1 = 2(-2)3 + 3(-2)2 12(-2) = 20 y 2 f ( x2 ) = f (1) 7 . ,
M1(-2, 12), M2(1, -7). . f ( x) 0
x (-2, 1), . f ( x) 0 x (-, -2)(1,
+), . , M1 , , ,
M2 , .
, f ( x) 12 x 6 , .
. M1(-2, 12) f (2) = 12(-2) + 6 < 0,
. M2(1, -7) f (1) 0 ,
. .


54

: IV
() .
.
:
1.
2. ()
a. - = 0
b. - = 0
3. ()
4. ( )
5.
6. .
.
.
() () ,
. () , .
. ()
.
3.3.2. f ( x)

x2
.
x 1

:
1. D(f) = R\{1}.
2. :

x2
0 x + 2 = 0 x = -2 - (0, -2);
x 1
02
x=0 y
= -2 - (0, -2).
0 1
y=0

3. . f ( x)

x2
( x 2)( x 1)
x 1

. , x1 = -2 x2 = 1.
, f(x) < 0 x (-2, 1), f(x) > 0 x [-2, 1].

x2
4. : = 1; = 1, lim
= lim
x
x x 1

2
x = 1. 4.
1
1
x


55

: IV
5. :

f ( x)

3
( f ( x) 0 -3 = 0
( x 1) 2

)

, .
(xD) f ( x) 0 f
.
6. f ( x)

6
.
( x 1) 3

x < 1,
. x > 1 f ( x) 0 ,
.
.
:
(xD) f(-x) = f(x)
(xD) f(-x) = f(x)

f ( x)

1
2
x 4

f ( x)

-.

x 3 3x
x2 1

O.

3.3.3.
1. ,

x3
y
x.
3

x3
y
x2 ,
3
2.

56

: IV

x3 x 2 ,

3 2
x 3,
4
y x( x 1) 3 ,

y x3

y x( x 1) 3 ,

y 3 ( x 1) 2 ,

y 3 (2 x 1) 2 .

3.

y x 4 3x 3 1 ,
4. f ( x)

y ( x 1) 4 .
x
x 4y = 0.
x 1
2

5. a b f ( x)

ax
P(2; 2,5).
x b
2

6.

y x2 x4 ,

x2
,
x 1
1

y 3 x 2 2x ,

ye ,

1
y arctg1 ,
x

y ln cos x .

x2

7. : ) f ( x) x 2 e x ; ) f ( x) xe x :
2

3.4.
,
. dy y = f(x) dy f ( x)dx .


57

: IV

dy

dy
dx , df ( x) f ( x)dx .
dx

dy/dx, . y = f(x), , dy dx .

4 (x) x0.
, ,
. 5 .
3.4.1.

1
2

: y x x = 49. = 1 = 7.
1

1
1
1
dy x 2 dx x 2 x 49 2 1 .
2
2
2

50 y y y dy = 7

4
5

1
1 99
.
49 2 7

2
14 14

: http://en.wikipedia.org/wiki/Differential_of_a_function
o : http://archives.math.utk.edu/


58

: IV

99 99 9801

50,0051 , .
14 14 196

3.4.2. ,

x , x = 100, = 10, = -2.

: y

dy

dx

1
.
10

98 y y y dy =
2 x 2 x 2 100
10 (1 / 10) 9,9 . 9,92 = 98,01 .
=

3.4.3. ,

127 .

: y 3 x , = 125, = 2, = 5.
2

1
1
1
dy x 3 dx x 3 x = (125) 3 2 ,
3
3
3

1
2
127 y dy y y = 5 (125) 3 2 5 .
3
75

3.4.4. sin 3,14.


: y = sin x, x = , x = -0,0015926 = 0.

dy cos xdx cos xx = (cos )(0,0015926...) ,


sin(3,14) y dy = 0 (cos )(0,0015926...) 0,0015926... .


3.4.5. tg( 0,75) .
: y = tg x, x = /4, x = 0,035398 y = 1. , dy

tg(0,75) y y = 1

0,03598...
cos

dx
0,035398...
=
,
2
cos x
2
cos
4

1,070796... .


59

: IV
6 3.4.6.
1. :
a.

b. ,
[A.

c. sin 31.
= f(17) f(16)+f(16)x =

x = 1

, sin 31 = sin 30 + cos 30

+
+

1 = 4+ ;

= 4+ ;

2. ln(x+2) ln(e+2).
[ f(x) = ln x x = (x+2)-x = 2,
f(x+2) f(x)+f(x)2 = ln x + .
ln(e+2) 1+ .]
3.
4.

= 10.

5. ln(x+1) ln x.
6.

7. cos(x+1) cos x.
3.5*. 7
(. The Mean Value Theorem MVT)
. ,
, .
. , ,
(Michel Rolle, 1652 - 1719),
, n- , .
.
3.5.1. f(x) (a, b) a
, b , f(a) = f(b) = 0, a b
c, a < c < b, f(c) = 0.

6
7

Garrett P: http://mathinsight.org/linear_approximation_differentials_refresher
4.4. .


60

: IV
: ,
y = f(x)
- (c, f(c)),
8 .
f(x) , . f(x) =
0, -,
(a, f(a)) (b, f(b)) .

(. secant),

, .

, .
, c f(x)
, .
f(a) = 0 f(x).
: ,
.
3.5.2. f : [a, b] R :
i.
ii.
iii.

,
(a, b),
f(a) = f(b),

c (a, b), f(c) = 0.


: [a, b]
, m M. m = M, f(x) ,
, . f'(x) = 0, . m M, c (a, b) f(c) = M
f(x) f(c) x [a, b]. , h > 0 k < 0

f (c h) f (c )
f ( c k ) f (c )
0
0.
h
k
f(c) 0, f(c) 0 (ii.) f(c) = 0.
. .
3.5.3. f(x) [a, b]
(a, b). c (a,
b)

Rolle's theorem: http://en.wikipedia.org/wiki/Rolle's_theorem


61

: IV

f (c)

f (b) f (a)
.
ba


(a, f(a))
(b, f(b)).
,
f(a) = f(b).
: .
, (a, f(a)) (b, f(b))
:

y ( x)

f (b) f (a)
( x a) f (a) .
ba

, g = f y
:
g (a) = f (a) y(a) f (b) y(b) = g (b) .
, c (a, b)
( f y)( x) f ( x) y ( x) =

f ( x)
f (c)

f (b) f (a)
. ( f y)(c) 0
ba

f (b) f (a)
.
ba

.
.
3.5.4. f(x) (a, b)
b , a b c,

f (b) f (a) (b a) f (c) , a < c < b.


.
, 9. f(x)
, c, AB.
(), , .
, , () .

:
http://sr.wikipedia.org/wiki/%D0%9B%D0%B0%D0%B3%D1%80%D0%B0%D0%BD%D0%B6%D0%BE%D0%B2%D0%
B0_%D1%82%D0%B5%D0%BE%D1%80%D0%B5%D0%BC%D0%B0


62

: IV

f (c) tg = tg

f (b) f (a)
,
ba

.
, .
3.5.5. f(x)
F(x) = f(x) + x,
. F(x) = f(x) + a b a
, b , f(x) x. F(a) = F(b),
f(a) + a = f(b) + b,

f (b) f (a)
.
ba

, c (a, b) F(c) = 0,
F(c) + = 0, .

f (b) f (a)
f (c) .
ba

63

: IV
.
.
3.5.6. A, B X f(x) a, b x.
h(x) AB.
h(x) |AB|, . B,
P(x) ABX.
:

x
2 P ( x) a
b

f ( x) 1
f (a) 1 .
f (b) 1

P(a) = P(b) = 0. , .
.

1
2 P ( x) 0
0

f ( x) 1

f (a) 1 a
f (b) 1 b

f ( x) 1
0
0

1 = f (a) f (b) f ( x)(a b) .


1

, P(x) = 0 f (c)

f (b) f (a)
.
ba

.
3.5.7. 10 f(x) = x3 3x2 + 2x [0, ].
a = 0, b = , c (0, ) f (c)

f ( x)

f ( 12 ) f (0)
. :
1
2 0

d 3
( x 3 x 2 2 x) 3 x 2 6 x 2 ,
dx

f (a) f (0) 03 3 0 2 0 0 ,
f (b) f ( 12 ) ( 12 ) 3 3 ( 12 ) 2 2 12

10

3
,
8

LAGRANGE'S MEAN VALUE THEOREM: http://youtu.be/cTp-VaCU7XU


64

: IV

f ( 12 ) f (0)
3
=> 3x 2 6 x 2 ,
1
4
2 0

f (c)

12 x 2 24 x 5 0 ,

x12
c 1

24 336
21
,
1
24
6
21
(0, 12 ) , . c (a, b).
6

. , f(a) = f(b),
f(c) = 0, .
, b = a + h,
f(a+h) f(a) = hf(c), .

f ( a h) f ( a )
f (c) .
h
h = x , ,
f = f(+h) f() . ,
= = + h h
c a a + h, . a + h
c a + h.
a < c < a + h, c = a + h, 0 < < 1,

f (a h) f (a) hf (a h) .

, .
. , m M f(x) (a, b),

f (b) f (a)
M.
ba

, f(x) = sin x, a = 0, b = h,

sin h
cos c , 0 < c < h.
h
0 < h < /2 cos h < cos c < 1,


65

: IV

cos h

sin h
1,
h

sin x
0.
x 0
x

, lim

. , , .
.
f(x) g(x) a b a b ,

f (b) f (a) (b a) f (c1 ) , a < c1 < b,


g (b) g (a) (b a) g (c2 ) , a < c2 < b.
x (a, b) g(x) 0. g(b)
g(a) 0, ,

f (b) f (a) f (c1 )

.
g (b) g (a) f (c2 )
.
, f(x) = x sin x, g(x) = x3, a = 0, b = h, 0 < h < .

c
c

2 sin 1
sin 1
1

cos
c
h sin h
1
2
2 = 1
=
=
2
3
2
c
6 2
3c 2
h
3c 2

2
2

, , .
, ,
. , c1 = c2
.
,

F ( x) [ f (b) f (a)] g ( x) [ g (b) g (a)] f ( x) .


F f g:

F ( x) [ f (b) f (a)] g ( x) [ g (b) g (a)] f ( x) .




66

: IV

F (a) [ f (b) f (a)]g (a) [ g (b) g (a)] f (a) = f (b) g (a) g (b) f (a) ,

F (b) [ f (b) f (a)]g (b) [ g (b) g (a)] f (b) = f (a) g (b) g (a) f (b) ,
. F (a) F (b) . , F : c
a b, F (c) 0 ,

[ f (b) f (a)] g (c) [ g (b) g (a)] f (c) 0 .


x (a, b) g(x) 0, g(b) g(a) = (b a)g(c) 0
,

f (b) f (a) f (c)


, c (a, b).

g (b) g (a) g (c)


c ,
. ,
(Cauchy's mean value theorem). , .
3.5.8. f(x) g(x) (a, b)
g(x) 0, b , c

f (b) f (a) f (c)


, c (a, b).

g (b) g (a) g (c)


: h( x) f ( x) g ( x) , f g ,
.

h(a) h(b) <=> f (a) g (a) f (b) g (b) , .

[ g (b) g (a)] f (b) f (a) .


h(x) h(a) = h(b), , c (a, b)
h(c) = 0.

0 h(c) f (c) g (c) , .


[ g (b) g (a)] f (c) [ g (b) g (a)]g (c) = [ f (b) f (a)]g (c) ,
.
3.6*.
(Guillaume Franois Antoine, Marquis de l'Hpital, 1661
1704.) (Analyse des infiniment


67

: IV
petits pour l'intelligence des lignes courbes, 1696.),
, .
f(x) g(x) , ,

( ). f(x)/g(x) f'(x)/g'(x),
.
, f(x) g(x) x = c,
g(x) 0 , lim
x c

lim f ( x) lim g ( x) , lim


x c

x c

x c

f ( x)
L . lim f ( x) lim g ( x) 0 ,
x c
x c
g ( x)

f ( x)
L.
g ( x)

lim
x c

f ( x)
f ( x)
.
lim
g ( x) xc g ( x)

6.3.1. f(x) g(x) (x0 h,


x0 + k) x0 g(x) 0, lim f ( x) 0
x 0

lim g ( x) 0 , , lim
x 0

x 0

f ( x)
f ( x)
, lim

x0 g ( x )
g ( x)
lim
x0

f ( x)
f ( x)
= lim
.
g ( x) x0 g ( x)

: (3.5.7.), c

f ( x) f ( x0 ) f (c)

, x0 < c < x.
g ( x) g ( x0 ) g (c)
, f(x0) = g(x0) = 0

f ( x) f ( x)
.

g ( x) g ( x)


68

: IV

x x0 + 0, c x0 + 0. lim

x x0 0

f ( x)
A ,
g ( x)

f (c)
f ( x)
A , . lim
A . , (x0 h, x0)
x x0 0 g (c)
x x0 0 g ( x)
f ( x)
f ( x)
. .
lim
lim
x x0 0 g ( x)
x x0 0 f ( x)
lim

, x 0 x c c,
, x . ,
x .
3.6.1. f(x) g(x) c, f(c) =
g(c) g(c) 0.

f ( x ) f (c )

f ( x ) f (c )
f ( x) 0
f ( x)
xc

= lim f ( x) .
= lim
= lim
= lim
lim
x c g ( x ) g (c )
x c g ( x ) g (c )
x c g ( x ) 0
x c g ( x )
xc g ( x )

xc

3.6.2. lim f ( x) 0 lim g ( x) 0 . x = , u 0,


x

1
1
lim f 0 lim g 0 .
u 0
u 0
u
u
,

1 1
1
1
f
f 2
f
f ( x)
f ( x)
u = lim u u = lim u =
= lim
.
lim
lim
u 0
x g ( x )
1 u 0 1 1 u 0 1 x g ( x)
g
g 2
g
u
u u
u
g ( x)
f ( x)
, lim
0 ,
x

c
g ( x)
f ( x)
g ( x)
f ( x)
g ( x)
lim
= lim
0 , lim
.
x c f ( x )
x c g ( x )
xc f ( x )
3.6.3. lim
x c

.
3.6.4.
i.

x2 x 2
( x 1)( x 2)
3
x2 x 2
2x 1 3

lim

1
.

2
2
x 1 2 x x 1
x 1 4 x 1
3
3
2 x x 1 ( x 1)(2 x 1)

lim


69

: IV

ii.
iii.
iv.
v.

sin 3x sin 3x 3
sin 3x
3 cos 3x 3
3

lim
.
.
x 0
x 0
4x
4x
3x 4
4
4
4

lim

ex
ex
ex

lim

lim
. .
x x 2
x 2 x
x 2
ln x
1/ x
lim x ln x lim
lim
lim ( x) 0 .
x 0
x 0 1 / x
x 0 1 / x 2
x 0
ln x
x ln x
1
x ln x x 1
x
= lim
= lim
=
lim

lim
x

1
x

1
x 1 x 1
x

1
x 1
x 1 x ln x
ln x
( x 1) ln x

ln x
x
1 ln x 1
lim
.
x 1 2 ln x
2
lim

3.6.5. .

2 sin x sin 2 x
,
x 0
x sin x

5x 4 4 x 2 1
,
x 1 10 x 9 x 3
ex x 1
. lim
,
x 0
x2

. lim

. lim

a x 1
,
x 0
x

. lim

1
x

x
. lim xe ,

. lim x .

3.7*. 11

y f (x)
(1, 2). 12 , - ,
, . . .
= (t) - t,
, :

dr
d2r
v
a
.
dt
dt 2

.
3.4.1. :

x ct , y

1 2
at ,
2

a c . , .
11

. : , I ,
, , , 1971.
12
,
, .


70

: IV
: ()

a 2
x .
2c 2

, .

x c , y at .
, x c .
, .

.

. , -

3.4.2. :

x b cos kt , y c sin kt .
, .

.
:

x
y
cos kt , sin kt ,
b
c

x2 y2
2 2 1 . , . ,
b
c
x bk sin kt , x bk 2 cos kt , y ck cos kt , y ck 2 sin kt .
, .
: x k 2 x ,

y k 2 y . ,
b2
c2
ck
bk
x , v x 2 y 2 = k 2 y 2 2 x 2 =
y , y ck cos kt
b
c
c
b
k
Xx Yy
c 4 x 2 b 4 y 2 . , 2 2 1 , X, Y
bc
b
c

x bk sin kt

, x, y .
, X = 0, Y = 0,


71

: IV

. d

b2c 2
c4 x2 b4 y2

1
x2 y2

b4 c4

b2c 2
k b 2 c 2 kbc
. v
, .

bc d
d
d

3.4.3. :

x be kt , y ce kt .
(), , . .
: (. )
xy bc . bc > 0, I III .
x bkekt , y cke kt . , ,
t, xy bck 2 . bc > 0, II
IV . v xi y j = bkekt i cke kt j = kxi k 2 y j .
, :

d r d r ds
v 0 ,
=
ds dt
dt

d 0 ds
d 0
d v dv
v2
a
0 v

n0 ,
=
= v 0 v
= v 0
R
dt
ds dt
dt
dt
(Frnet)

d r d 0 n 0

,
.
ds ds
R

vt v , vn 0 , vb 0 , at v , a n

v2
, ab 0 . ,
R

at
2

1
1
2
(v a ) 2 , a n 2 (v a ) 2 .
2
v
v

v 2 k 2 ( x 2 y 2 ) , a v k 3 ( x 2 y 2 ) , a v 2k 3 xy k ,

72

: IV

at
2

k 4 (x 2 y 2 )2
1
6
2
2 2
=
,

k
(
x

y
)
x2 y2
k 2 (x2 y 2 )

an
2

4k 4 x 2 y 2
1
6 2 2
=
.

4
k
x
y
x2 y2
k 2 (x2 y 2 )

.
3.4.3. :

x 2 y a sin t
y 2 x a cos t ,
a . , x = y = 0 t = 0.
: ,
x 2 y a cos t , ,
x 4 x 3a cos t .
. xh C1 cos 2t C2 sin 2t ,
. ,
x p A cos t B sin t , . .
, A = -a, B = 0,

x C1 cos 2t C2 sin 2 y a cos t .


1
2

y y ( x a sin t ) ,
x

y C1 sin 2t C2 cos 2t a sin t .


C1 C2 ,

x a(cos 2t cos t ) , y a(sin 2t sin t ) .


.
, .
x1 x a , y1 y ,
. ,

r a(2 cos 1) .

73

: IV
( ).


74

: IV

4.
. ,
f(x) g(x), f ( x) g ( x) , f(x)
g(x) ,
,

g ( x)dx = f(x) + C,
C . ,
x2 2x,

2 xdx = x

+ C,

C R (x + C) = 2x.
2

g(x),
b

g ( x)dx ,
a

Oxy ,
-.
b

g ( x)dx f (b) f (a) , f g.


a

4.1.
4.1.1. f(x), (a, b).
F(x) , F(x) = f(x), dF(x) = f(x)dx,
x , f(x).
.
, 3.2.1.
() .
4.1.2.
I.
II.
III.
IV.
V.
VI.

x 1
x dx 1 C , -1, x > 0.
1
x dx ln | x | C , x 0.
ax
x
a dx ln a C (a > 0, a 1) ,

dx e x C .

cos xdx sin x C .


sin xdx cos x C .
dx

cos

tg x C , x

k , k Z.

75

: IV

dx

VII.

sin x ctg C , x k, k Z.

VIII.

IX.

arcsin x C
, -1 < x < 1.

1 x 2 ar cos x C
dx

arctg x C
dx

1 x 2 arctg x C .

()
.
X.

XI.

XII.
XIII.

dx
1 x
dx

Arc sh x C ln( x 1 x 2 ) C .
Arc ch x C ln( x x 2 1) C .

x 1
dx
1 1 x
1 x 2 Arc th x C 2 ln 1 x C , |x| < 1.
dx
1 x 1
1 x 2 Arc cth x C 2 ln x 1 C , |x| > 1.
2


. ,
,
.
4.1.3. .
1. c R f(x)
:

cf ( x)dx c f ( x)dx , c = const.


2. () () :

[ f ( x) f
1

( x)]dx f1 ( x)dx f 2 ( x)dx .

3. , x = (t):

f ( x)dx = f [ (t )] (t )dt .
4. :


76

: IV

udv uv vdu .

: 1. F(x) = f(x), cF(x) = cf(x). 2. F1 ( x) f1 ( x) F2 ( x) f 2 ( x) ,

( F1 ( x) F2 ( x)) = F1 ( x) F2 ( x) . 3. x = (t)

dx d (t )
=
= t (t ) ,
dt
dt

f ( x)dx f ( (t ))tdt . 4. u = u(x) v = v(x),


d (u v) u dv v du , d (u v) u v = u dv v du , udv uv vdu .
4.1.4.

x 51

x6

C1 =
1. 3x 5 dx = 3 x 5 dx = 3
C , C1 , C = 3C1.
5

1
2

2.

(5x

3x

4
x

1
2

)dx = 5 x 3 dx 3 xdx 4 x dx =

1
2

5x
3x
4x

C.
1
4
2
2

3. sin nxdx = [: nx = t, : x = t/n, dx = dt/n]

= sin t

cos nx
C .
n

=
4.

1
1
1
dt = sin tdt = cos t C = [ ]
n
n
n

x cos xdx = xd sin x = x sin x sin xdx = x sin x cos x C .

, ,
.
4.1.5.

(2 x 5 x 3 x ) 3
5

x7

dx =

8 x 2 60 x

1
3

8 x 3 3 4 x 5x

x 3 2 x 25 x 2 3 x 2 125 x 3 x
5

150 x
x

7
5

1 2
2
2 3

125 x 4

x7

dx

dx


77

: IV
3 7

7 7

19 7

= 8 x 2 5 dx 60 x 3 5 dx 150 x 6

dx 125 x

7
5

dx

80 10
900 15 14 4500 2 30 23 625 3 5 3
x x
x x
x x
x x C .
11
29
83
18


sin 2 x

1 cos 2 x
1 cos 2 x
cos 2 x
, .
2
2

4.1.6.

sin

xdx =

1
1 cos 2 x
dx =
2
2

dx cos 2xdx = 2x sin42x C .

.
4.1.7.

x
x
x

2 dx = 3 sin 2 d 2 = 3 cos 2 C .
3
3
3

i. sin

2
1

ii.

iii.

= 4
3

dx

4
3

dx
k x
2

1
k

x
x 3

d4
4
3
3

= 3
1
2

1
3
3

2
3

dx
x
1
k

= [:

x
t]=
k

9
x
C = 4
5
3

dt
1 t

= arcsin

5
3

C .

x
C .
k

(iii.) .
4.1.8.
a.

b.

dx
a bx 2

dx
1
= 2
2
x
k

dx
2

a
2

b x

dx

x
1
k

= [:

1
b

arcsin

x
a
b

C =

b
x ab
arcsin
C.
b
a

x
dt
1
x
= arctg C .
t]=
2
k
k
k
1 t


78

: IV

c.

dx

k x
2

1
k

dx
x
1
k

= [:

x
t]=
k

dt
1 t

= ln(t 1 t 2 ) C1

x
x k 2 x2
x 2

C1 = ln( x k 2 x 2 ) ln k C1
= ln
1 2 C1 = ln
k

k
k

= ln( x k 2 x 2 ) C .
, .
4.1.9.

dx
1
= 2
2
x
k

dx
x
1
k

= [:

1
x
dt
t]=
k 1 t2
k

1. [ |t| < 1] =

1
kx
1 1 1 t
=
ln
C ,
ln
k 2 1 t 2k k x

2. [ |t| > 1] =

1 1 t 1
1
xk
=
ln
ln
C .
k 2 t 1 2k x k

(a 0) ax 2 bx c =
2
2
2
2 b
b
b 2 4ac
b
b b

, x
a x x c = a x
t ,
a
2a
2a
2a
2a 2a

.
4.1.10.

2
3

arctg

2x 1
3

dx
dx
=
=
2
x 1
1
3

x
2
4

dt
3
t
4

dt
3

t 2

2
3

arctg

2t
3

C =

C .

, ()
.


79

: IV

4.1.11.

dx
=
x2

1 ( x 1) ( x 2)
1 dx
1 dx
=
dx =

3 x 2 3 x 1
( x 1)( x 2)

( x 1)( x 2) dx = 3

1
1
1 x2
ln | x 2 | ln | x 1 | C = ln
C .
3
3
3 x 1
,
.
4.1.12.

(arcsin x) 5
1 x

dx = [: arcsin x = t, dt

dx
1 x2

t6 1
= t dt =
= (arcsin x) 6 C .
6 6
5

4.1.13.

sin xdx .

sin xdx = I(x). I ( x) sin xde x = e x sin x e x d sin x

= e x sin x e x cos xdx

= e x sin x cos xde x

= e x sin x (e x cos x e x d cos x)

= e x sin x e x cos x e x sin xdx .


, I ( x) e x sin x e x cos x I ( x) , . 2I ( x) e x (sin x cos x) ,

ex
I ( x) (sin x cos x) .
2

4.1.14. .
1.

2x3
1
2
5 x 3x 2 dx .

4
3
(3x 2 x x 2)dx ,

2.

80

: IV

1
2
dx ,

x 5 x 3

x2

x5

3
4
dx .

x
x3

3.

x 2
1

x 3

x 3 dx ,

x 2 dx .

4.

ax b

17

dx ,

dx ,

px q

sin(kx n)dx ,

dx .

5.

cos
tg

sin
ctg

xdx ,

xdx ,

xdx ,

xdx .

6.

dx

2x 3

3 5x 2
dx

dx

5 3x

dx

1 4x 2

,
.

7.

ctg xdx ,

tg xdx ,
cos 7 x

sin 5 x

sin 7 x

dx ,

cos 5

dx .

8.

xdx

1 x
xdx

2 3x 2

xdx

1 x

xdx
3

3 2x 2

9.

dx
,
x 1
dx

4x x 2

dx

15 6 x 9 x

dx
x 2 6x 2

,
.


81

: IV
10.

dx
,
x2
2x 1
x 2 5x 6 dx ,

dx
,
5x 6
2x 1
x 2 3x 2dx .

10.

x e
2

ln xdx ,

dx ,

arctg xdx ,

cos xdx .

11. ( ) 13: wiki.elemenat.com.


4.2*. 14
, , ,
, , .
.
4.2.1. F R3 R.

F ( x, y( x), y ( x)) 0 ,
y : [a, b] R ,
, y(x).
4.2.2. y ( x) y( x) 2 1 .
2

y1 ( x) cos x () ,
, . ,
y( x) sin( x C ) , C y ( x) cos( x C )
. , y = 1,
, .
4.2.3. F n+1 . n
F ( x, y, y ,..., y ( n ) ) 0 , y ( k )

dky
, k = 1, , n .
dx

F n+1 .
4.2.3.

y 2 y 3 .

13

http://www.elemenat.com/cyr/wiki/index.php?title=_
Diferencijalne jednadbe: http://youtu.be/2VOMDvqpxWw
What is a differential equation: http://youtu.be/-_POEWfygmU
14


82

: IV

y y y x y .

4.2.4. :

x2 y2

1.
a2 b2

x 2 y 2 C1 x C2 y C3 0 ,

: () C1, C2, C3 ,

2 x 2 yy C1 C2 y 0 , 2 2( y ) 2 2 yy C2 y 0 .
C2 , x.

1 ( y ) 2 yy

0 , 3 y ( y ) 2 (1 ( y ) 2 ) y 0 .
y

b2
yy
2 x 2 yy
() 2 2 0 2
.
x
a
b
a
b2
b2 2 , xyy y 2 b 2 .
a
yy xy 2 xyy 0 .
()
.
4.2.5. .
i.

y ( x)

3 c
2 xy 4 y 3 .

4 x2

3
2

4 x 2 y 12 xy 3 y 0 , x > 0.

ii.

y ( x) x

iii.

(1 e y ) y 1 () y e y x C .

.
( )
, .
y f (x) , f .
y

f ( x)dx C .

f ( x)dx g ( y)dy 0 ,
. , , .

f ( x)dx g ( y)dy C . ,

F ( x) G( y) C , C .

83

: IV
4.2.6. :
i.

dy
6x 2 y ,
dx

: i.

ii. y

dy
5 x 2 dx ,
y

x 2 2x 3
,
3y 4

iii. y e y (2 x 4) .

3
x3
1
2
,

ln
y

C1 , y Ce 2x ,
dy

5
x
dx
y

C e C1 , C1 . ii. (3 y 4)dy ( x 2 2 x 3)dx ,


3

y2
x3
4y
x 2 3x C .
2
3

, () C. iii.

y( x) ln( x 2 4 x C ) , C 4.

:

y f ( x, y) , y( x0 ) y0 .
.
4.2.7.

(1 e x ) yy e x , y(0) 1.
: ydy

12
y2
e x dx
x
,

ln(1 e 0 ) C , .

ln(
1

e
)

C
x
2
2
1 e

e
1
ln 2 -0,193. y ln (1 e x ) 2 .
4
2

y
. y = ux, u
x
. y u x u u x u f (u) , f(u) u x 0,
y f


ln | x |

du
dx
,

f (u ) u
x

du

du
C . x C1e f (u )u , C1 e C
f (u ) u

.
4.2.8. )

dy 2 x 5 y
.

dx
2x y


84

: IV

ax by c

.
) y f
Ax By C
: ) z

2 x 5 xz 2 5 z
y
xz z
=
,
2 z
2 x xz
x

1 2 5z

z . , :

x 2 z

4 ln | z 2 | 3 ln | z 1 | ln | x | C
4 ln | y 2 x | 3 ln | y x | C

yx
z 1

y 2 x.
z 2,

( y x) 3 C1 ( y x) 4 .
) :
1.

A B

0 , y = u + , x = v + , ,

av bu a b c
du
. ,
f
dv
Av Bu A B C
a b c 0
du
av bu
,
f
=

dv
Av Bu
A B C 0
u

ab
v , .
f
u

A B
v

2.

A B

0 , A = ka, B = kb, k . u = ax + by
uc
, . .
ku C

u a bf

y f ( x) y g ( x) , f g .
g , .
y f ( x) y 0 , y 0,
f ( x ) dx
dy
f ( x)dx , . ln | y | f ( x)dx C , y C 2 e
, C 2 e C1
1
y

. , C2 = 0 () y = 0.

85

: IV
, .
C2 , x.
f ( x ) dx
f ( x ) dx
f ( x ) dx
, y C ( x)e
.
y C ( x )e
C ( x) f ( x)e
f ( x ) dx
f ( x ) dx
, C ( x)e
,
g ( x) , . C ( x) g ( x)e

C ( x) C3 g ( x)e

f ( x ) dx

dx . ,

f ( x ) dx
f ( x ) dx dx , C
y e
3
C 3 g ( x )e

.
()
y C3 F1 F2 .
4.2.9. y

1
y x.
x

1
1
1
x3 C x3
x dx
x dx

: y e
, C
dx , . y C =
C xe
x
3 x 3

.
4.2.10.
1. y C1e x C2 e 2 x .
[ y y 2 y 0 ]
2. y 5 y y(0) = 10.
3. xy y 2 1 , y(1) = 2.
[y

1 Cx 2
1
,C ]
2
3
1 Cx

4. . y 2 y 1 y 2 .
[ y arctg y x C ]
5. y 1 x 2 y 2 x 2 y 2 , y(0) = 1.

x3
dy

tg
x

C C ]

(
1

x
)
dx
2

3
4
1 y


86

: IV
6. y y tg x cos 2 x , y(0) = 2.
[ y (C sin x) cos x , C = 2]
7. y

2 xy 2
, y(0) = 0,4.
1 x2
[ y (1 x 2 )(arctg x

x
C ) , C = 0,4]
1 x2

8. 2 xy y x 1 , y(2) = 4.
[ 2x, y x 1 C x , C

3
2

4.3.
P Oxy
y = f(x) -, x = a x = b, a < b.

ba
, n
n
. x0 a , x1 a n , x2 a 2 n , ..., xn b .
[a, b] n n

D.
,
n

ba
, mi f (ci ) , M i f (Ci ) , ci , Ci [ xi 1 , xi ] (i = 1,
n

2, , n), .
[a, b] n = 5
5

ba
. i = 1, 2, , n,
5

ci Ci [xi-1, xi]
, f (ci ) f (Ci )
. ,
,
s n

f (c )
i 1

S n f (Ci ) n . P
i 1

s n P S n .
d(f,D)


87

: IV
(), g(f,D)
(). , ()
I * sup{d ( f , D)} , () I * inf{g ( f , D)} .
, f a b I I * I * ,
.
b

, I

f ( x)dx , a b [a, b]
a

, f , .
.
4.3.1. , I III , . f(x) = x. P
-, [a, b].
(a, b) n

ba
ba
. , x k a k
, f ( xk 1 ) xk 1 =
n
n
n

a (k 1) n , s n a (k 1) n n =
k 1

na (1 2 ... (n 1)) n n = na n(n 1) b a n

= ... =

ba
ba
a b
. n ,
2
n

b2 a2
b2 a2
, . xdx
.
2
2
a
b

n 0 , P

.
i.

f ( x)dx f ( x)dx ,

ii.

f ( x)dx 0 ,
a

iii.

iv.

cf ( x)dx c f ( x)dx ,

:
b

v.

f ( x) g ( x)dx f ( x)dx g ( x)dx ,


c

f ( x)dx f ( x)dx f ( x)dx ,



88

: IV
b

vi.

: (x[a,b]) f(x) 0

f ( x)dx 0 ,
a

vii.

: (x[a,b]) f(x) g(x)

f ( x)dx g ( x)dx ,

a
b

viii.

a
b

f ( x)dx f ( x) dx .

-
. .
4.3.2. f : [a, b] R [a, b]
F : [a, b] R x (a, b). - :
b

f ( x)dx F (b) F (a) .


a

. D [a, b] n . i = 1, 2, , n
F [xi-1, xi] (xn-1, xi).
( f

f (b) f ( z )
) i
ba
(xi-1, xi) F ( xi ) F ( xi 1 ) = F ( i )( xi xi 1 ) = f ( i )( xi xi 1 ) .
f ( )

inf{ f ( x) : x [ xi 1 , xi ]} = mi f ( ) M i = sup{ f ( x) : x [ xi 1 , xi ]}

mi ( xi xi 1 ) F ( xi ) F ( xi 1 ) M i ( xi xi 1 ) , i = 1, 2, , n.

n

m( f , D) F ( xi ) F ( xi 1 ) M ( f , D) ,
i 1

m( f , D) F (b) F (a) M ( f , D) .
D [a, b]


89

: IV

I * F (b) F (a) I * .
f I * I
*

f ( x)dx , f ( x)dx F (b) F (a) ,

.
,
, .
b

f ( x)dx F ( x) | .
a

4.3.3.
b

) f(x) = x (4.3.1)

xdx
a

b
x2
b2 a2
C| =
,
2
2
a

.
, .
) f(x) = sin x -,

, [0, ], sin xdx cos x | = cos cos 0 = -(-1) + 1 = 2.


0

x3 1 1
| .
) 1 f(x) = x x dx
3 0 3
0
2

4.3.4. y = f(x), x-, x = a x = b


c

P f ( x)dx
a

f ( x)dx + f ( x)dx ,

, f1(x) f2(x), (x[a, b]) je f1 ( x) f 2 ( x) ,


b

x = a x = b, P

f ( x) f
1

( x)dx .


90

: IV
4.3.5. y1 x 2 2 x 3 y 2 0,5x 2 x .
: A B .
y1 y 2 1,5x 2 3x 3 0 ,

x 2 2 x 2 0 .
2,732
x12 1 3
. 1,
- 0,732
A(0,732;1) B(2,732;1) ,
, :
2 , 732

2 , 732

(y

y 2 )dx =

3x 3)dx =

0 , 732

0 , 732

(1,5x

2, 732
1,5 x 3 3x 2

3x | = 10,392.
3
2
0 , 732

, P 10,4.

4.3.6. : (i) x 2 y 2 r 2 , (ii)

x2 y2

1.
a2 b2

: (i) r.
, y

r 2 x 2 0 x r . ,

Pk 4 r 2 x 2 dx = (: x r sin t , dx r cos tdt )


0

=4

1 cos 2t
dt
2
0

r 2 (1 sin 2 t )r cos tdt = 4r 2 cos 2 tdt = 4r 2


0

2
1
d
(
2
t
)

2
2
= 2r 2 dt cos 2t
= 2r t sin 2t | = r .
2
0
2

(ii) , I .


91

: IV

x2
y b 1 2 , I
a
.
a

Pe 4 b 1
0

x2
dx = ( x a sin t )
a2

= 4 b cos 2 t a cos tdt


0

1
2
= 4ab cos tdt = 2ab t sin 2t | = 2ab .
2
0
0
2

y = f(x) ,
()
, ds2 = dx2 + dy2.
, dx dy , ds
, . .

ds
dy
1 ,
dx
dx

dy
y (x) , ds 1 ( y ) 2 dx . y = f(x).
dx

s :
b

s 1 ( y ) 2 dx .
a

y ,

x f 1 ( y) , , ( = f(a), = f(b)):

s 1 x 2 ( y )dy .

4.3.6. y = ch x, x = 0 x = 1,12.


92

: IV

e x ex
,
2
e x ex
,
y sh x
2

y = ch x =

1 y 2 = 1 sh 2 x = ch x ,

1,12

1,12

ch xdx = sh x

= sh 1,12 = 1,37.

.
ch x

e x ex
e x ex
sh x
,
2
2

ch 2 x sh 2 x 1, ch 2 x sh 2 x ch 2 x , 2 sh x ch x sh x , (ch x) sh x , (sh x) ch x
.
15.
. , ,

( ) .

- y = f(x)
-. :
b

S 2 f ( x) 1 f 2 ( x)dx .

V f 2 ( x)dx ,

15

( 20. 21.):
http://www.elemenat.com/cyr/wiki/index.php?title=_


93

: IV
4.3.7. f ( x)

x x [1, 2] -.

.
:
:
b

V f ( x)dx =
2

x dx 4,7124 .
2

:
b

S 2 f ( x) 1 f 2 ( x)dx =
a

= 2

1
x 1
dx
2 x
= 8,2832 .

4.3.8.
1. -

2 x

3x dx ,

1
dx ,
x2

x 23 x dx ,

x 2 3 x dx .

2.
2

x x
3

dx ,

x 1
x

dx ,

x2 x 6
1 x 2 dx ,

x 2 3x 10
0 x 2 dx .

e x ex
2 dx ,
1

e x ex
1 2 dx ,

3.
3

x 1
2 x 1dx ,

x 1

x 1dx .
2

4. c

f ( x)dx f (c)(b a) ,
a


94

: IV

f ( x)

1
1 x

f ( x)

, a = 0, b = 1,

1
, a = 0, b = 1.
1 x2

5.

cos( x a)dx 0 [0, ].


0
a

[ sin( x a) | = sin 2a sin a = 0 sin a(2 cos a 1) 0 ]


0

6.


a 2

x 1

1
dx 4 ?
x
[a a 4 2

a 3 0 a 9]

7. x = a x = 2a, a > 0,

y 1

1
. .
x2
2a

[ f (a)

ydx = x
a

f (a) 1

x 1 2 a
1
,
| = 1
1 a
2a

2
1
= 0 => a
]
2
2
2a

8. 4.3.1.
9. :
) : x2 + y2 2x + 2y 2 = 0; ) : 9(x 1)2 + 16(y + 1)2 = 144.
10. y 2 2 px x 2 2 py .
11. :

y 3x 2 1 y x 2 1 ;
2
y x 2 2x 1 y ;
x

y x2 1 y x 1;
y

x2
1
y

.
2
x2 1

12. r.
13. 9 y 2 4 x 3 x = 0 x = 3.
2
3

2
3

14. x y 1 .

95

: IV
15. y x 2 9 .
16. :
a)

2 y 3x 2 x 2 y 2 6 x 0 , -.

b)

y x 2 2 y | x | , -.

c)

x 2 y 2 1 y 2 4 x 0 , -.

d)

y x 3 , y 8 x 0 , -.

17. .
4.4*. 16
() 4.4.1. f c (a, b) D
(a, b). c, f (c) 0 .
: f c (a, b). f
, . f (x) , x = c

. :

f (c ) lim

x c 0

f ( x) f (c) ()

0
xc
()
,

f (c ) lim

x c 0

f ( x) f (c) ()

0
xc
()

,
,
f (c) 0 .
. .

, .
, [a, b]
b .
.
4.4.2. f x D x (
f (x) ), f ( x) 0 .

16

3.5. , .


96

: IV
() 4.4.3. f [a, b],
(a, b) f(a) = f(b). c (a, b)
f (c) 0 .
: , f [a, b], f ( x) 0 ,
. , f ,
c (a, b) .
() 4.4.4. f, g : [a, b] R, g(x) 0,
(a, b) . c (a, b)

f (c) f (b) f (a)


.

g (c) g (b) g (a)


: g(x) 0 x (a, b) g(a) g(b). ,
g(a) = g(b), x (a, b) g ( x) 0 .
F ( x) f ( x) f (a)

f (b) f (a)
g ( x) g (a) , ,
g (b) g (a)

. , DF D f Dg F (a) F (b) 0 .
f g ,
F. , F c (a, b)
F (c) 0 . 0 F (c) = f (c)

f (b) f (a)
g (c) , .
g (b) g (a)

g ( x) x , g ( x) 1 g (b) g (a) b a ,
.
() 4.4.5. f, g : [a, b] R, g(x) 0,
(a, b) . c (a, b)

f (c)

f (b) f (a)
.
ba


97

: IV
,
.

f (b) f (a)

ba


A(a, f (a))
B(b, f (b)) , f (c)

C (c, f (c)) .


.


c (a, b) ,

.
, f
, . , f
, , c
. , f ( ),
, c .


98

: IV
( ) 4.4.6. f, g : [a, b] R, g(x) 0,
b

f (c )

c (a, b)
g (c )

f ( x)dx
a
b

g ( x)dx

1
f ( x)dx .
. ( g ( x) 1 ) f (c)
b a a

: F G f g, . f g
, F G , G( x) g ( x) 0 . F G
, c (a, b)
b

f (c) F (c) F (b) F (a)


=
=
=
g (c) G(c) G (b) G (a)

f ( x)dx
a
b

g ( x)dx
a

. g ( x) 1 .

, .
f(x) , a
b,
b - a f(c).
. f(c)
[a, b].
4.5*.
, :

x x(t ) , y y(t ) , t1 t t 2 .
b

dx x(t )dt , P

ydx
a

t2

P y (t ) x (t )dt ,
t1

t = t1 x = a, t = t2, x = b.
4.5.1. .


99

: IV
. -.
r = 2.

x r (t sin t ) , y r (1 cos t ) ,

0 t 2 . , x(t ) r (1 cos t )
2

P r (1 cos t )r (1 cos t )dt = r 2 (1 cos t ) 2 dt =


2
2
2

= r dt 2 cos tdt cos 2 tdt .

0
0
0

( ) ,

t sin 2t
Pr t
2
4

= r 2 (2 0) ,

P 3r 2 .
, ,
.

dP

1 2
r d ,
2

1 2
r ( ) d .
2

OAB, AB r r ( )
- OA OB ,
.


100

: IV
4.5.2. r 2 a 2 cos 2 .
: a = 2 . r ( ) cos 2



2 cos 1,68 , r 2 cos 0 ,
4
2
8
4

: r (0) 2 , r

3
3
r 2 cos
2 0,71 - , ... . ,
4
8
( x 2 y 2 ) 2 a 2 ( x 2 y 2 ) ,
, , -.
.
,
.
.
,
.

a 2 sin 2
P a2
=

cos
2

2
2
4
2
=

a2
2

4
0

2
1
a
,
0
2
4

P a 2 .
4.5.3. r a
2

1
a2 3
P a 2 2 d =
2 0
2 3
4
P 3 a 2 .
3

=
0

a2
8 3 , .
6

,
- Op O,
P -.
4.5.4. ( ) r a(1 cos ) .
:


101

: IV
2

1
a 2 (1 cos ) 2 d
2 0

a2
=
2
=

(1 2 cos cos

)d

a2
sin 2
2 sin
2
2
4

a
(2 ) , .
2
3
P a 2 .
2
=

(. )

.
4.6*.
() :
, , ,
( ), (
) .
4.6.1. x = a x = b
= F . 17 (. work) W = F(a b).
,
, . F = F(x) x [a, b], [a, b]
x0 = a, x1, , xn = b. n N xk = xk-1 xk, k = 1, 2, , n. xk
, , F(x)
F(tk). , Wk = F(tk)xk [xk-1, xk].
[a, b] [xk-1, xk]
, W F(x) x = a x = b
n

k 1

W lim F (t k )x k = F ( x)dx .
n

-.

17


102

: IV
,

= Fx + Fy + Fz = x + y + z ,
,


C.

W F dr .
C


= Fxdx + Fydy + Fzdz,

W Fx dx Fy dy Fz dz .
C

, xy ,
=
= 0.
, .
4.6.2. m v0.
= m .
:
,
t :

y = 0,
.

gt 2
0 ,
2
2v sin
gt

t v0 sin t 0 t 0
.
2
g

v0 sin t

= m = m(0, -g). P = Fx,


Q = Fy,


103

: IV

. ,
,

Fy
x

Fx
0 , Fx = P Fy = Q.
y

f ( x, y )
f ( x, y )

f(x,y)
x
y
, .

u
d
Q( x, y ) g ,
C1 ( y ) g C1 ( y) gy C2 .
y
dy

C , u(x,y) = -gy.
O(0,0) A(L,0)
W = u(A) u(O) = 0.
.
4.6.3. ,
. ,
k .
,


F kx .

, .


104

: IV
x

A kxdx
0

kx2
.
2

, .
. ,
. .
, .
4.6.4. 20 cm. 40 N
30 cm. 35 cm 38 cm?
: , F(x) = kx, k > 0 . , x
. ,
x = 30 cm 20 cm = 0,10 m,
0,35 0,20 0,38 0,20 .
, 40 = 0,10k, k = 400, F(x) = 400x

0 ,18

0 ,18

2
400xdx = 200x | = 1,98 J.

0 ,15

0 ,15

35 cm 38 cm 1,98 (J = Nm = kg m2/s2).
.
4.6.5. 15 4
, 12 .
? 1000 kg/m3.
: = 0 , = 15 . [0, 12] x


105

: IV
n . ,

ri x.
.
ri : xi* 4 : 15 , . =
.

= () ,
(1000)
x =
2

4
= 1000 xi* x , tj.
15
2
640
mi
xi* x .
9


= =(9,8) ( )2x,

15 . i-

Ai Fi (15 xi* ) = (9,8)


A

640
( xi* ) 2 (15 xi* )x .
9

A ,
i 1

A lim (9,8)
n

i 1

640
( xi* ) 2 (15 xi* )x , .
9

12

12

640
640 2
A (9,8)
(15 x 2 x 3 )dx
x (15 x)dx = (9,8)
9
9
0
0
= 9,8

12
640
(5 x 3 14 x 4 ) | = 7 566 362,543 J.
9
0

,

. .
.
4.6.6. l M .
m d .
.


106

: IV

(. infinite) ,
,

, , .
.

_1 l

_1 l
2

M
d

m
,
(dM) ,
(x, , r).
, . (.
magnitude)
dF m dM

dF

mdM
,
r2

= 6,6738410-11 m3kg-1s-2 .
,
, . , ,
d , .

F dF .
, ,

Fx dFx dF sin

mdM sin
r

Fy

G mdM cos
.
r2

, , . Fx = 0,
. ,

F Fy

mdM cos
r2

. r
,

r 2 d 2 x 2 cos

d
d

.
r
d 2 x2
M
dx .
l

dx dM, pa je dM
l/2 + l/2,

l / 2

Mmd
dx

2
l l / 2 d x 2

3/ 2


107

: IV
.
() : ,
, ( ),
( ), .
4.6.7. C .
(x,y,z).

m ( x, y, z )ds ,
C

ds dx 2 dy 2 dz 2 ,
dx, dy, dz,
OXYZ. C r (t ) ( x(t ), y(t ), z (t ))
,
2

dx dy dz
m ( x(t ), y (t ), z (t )) dt .
dt dt dt
a
b

C xy ,

m ( x, y )ds ,
C

dx dy
m ( x(t ), y (t )) dt .
dt dt
a
b

4.6.8. C, (1,1) B(2,4). ,


(x,y) = 3x + 2y.
: AB:

x xA
y yA

t ,
xB x A y B y A
x 1 y 1

t , .
2 1 4 1

x 1 y 1

t ,
1
3

108

: IV

x t 1

y 3t 1
t [0, 1].
2

dx dz
dx dy
m ( x, y, z ) dt = [3x(t ) 2 y (t )] dt =
dt dt
dt dt
0
a
b

d (t 1) d (3t 1)
2
2
0 [3(t 1) 2(3t 1)] dt dt dt = 0 (9t 5) 1 3 dt = 10 0 (9t 5)dt =
2

9t 2
1 19 10
10
5t | =
30 .
2
2
0
4.6.9. x2 + y2 = 1 A(1, 0) B(0, 1).
(x,y) = xy.
:
x cos t , y sin t ,
t [0, /2]. ,
:

dx dz
m ( x, y, z ) dt
dt dt
a
2

dx dy
0 x(t ) y(t ) dt dt dt
2

dx dy
= cos t sin t dt
dt dt
0
2

cos t sin t sin t

2
12
1
cos t dt = cos t sin tdt = sin 2tdt = ( cos 2t ) | =
20
4
0
0

1
1
( cos cos 0) .
4
2
4.6.10. C
(x,y,z).

M yz
m

, y

M xy
M xz
, z
,
m
m



109

: IV

M yz x ( x, y, z )ds , M xz y ( x, y, z )ds , M xy z ( x, y, z )ds


C

. .

I x ( y 2 z 2 ) ( x, y, z )ds ,
C

I y ( x 2 z 2 ) ( x, y, z )ds ,
C

I z ( x 2 y 2 ) ( x, y, z )ds .
C

4.6.11. r = a(1 + cos ), = 1, = 1.


:

= 0. ,
,
.
:
2

dr
m (r , ) r d
d
a
b

2
2
= 1 (1 cos ) ( sin ) d
0

1 2 cos cos 2 sin 2 d

2 2 cos d

2 1 cos d =
0

2
0

2 cos d = 4 cos d = 4 sin | 4 .


2
2 2
20

0
2

My
2

dr
M y F ( x, y )ds = F (r cos , r sin ) r
d .
d
C
a
b


110

: IV

/2 = z

My
m

16
5

4
.
5

, (0, 4/5) .
4.6.12. Ix x2 + y2 = a2, = 1.
:

Ix -


111

: IV

, Ix = a3.
.
4.6.13. C
()
, .

0 = 1,2610-6 H/m
. .
4.6.14.
r ,
I.
: r
,
.
,
Bdr.

2rB 0 I , . B

0 I
.
2r

4.6.15.

112

: IV
S
C
.
,
C,
:


1831,
.
4.6.16. E

900 km/h.
: . ,
t 2rBx 2rBvt , v , x = vt
, B .

, .

E d r . ,
C

.
. ,


113

: IV

0,00025
= 0,004 V/m.

2r 2 0,01


114

: IV

5.
. ,
,
.
. ,
, , 1700.
. , (. . )
, ,
. ,
, .
, ,
.

: -, - . ,
, .

6. 4. .... ,
1002000000000.
.
(202. 220. . ...) (
), ( ) .
, ,
.
(, 100. .
...) . , ,
18. , :
.
, ,
500 .. 654321 2,
19.
,
( 1150. ..).
(Musurgia Universalis, 1650.) ,
,
.

18
19

Solomon Gandz, Studies in Hebrew Astronomy and Mathematics, New York, Ktav, 1970, . 494-496.
G. Chakravarti, Bull. Calcutta Math Soc., 24 (1932), 79-88.


115

: IV

(Trait du trianglearithmtique, 1665.),
, . ,
.
(960 1279.) .
(- -, 1303.)
-, .
.
5.1.
. S {a1 , a2 ,..., an }
k, 1 k n. (a1 , a2 ,..., ak ) k-.

:
i.

k- S k-
S. Vkn .

ii.

k- S k-
S. Vkn .

5.1.1. S = {1, 2, 3, 4}.

V24 ( )

V24 ( )

5.1.2.
, ?
: , n = 2, k = 1, 2, 3, 4, 5.
, V12 2 , .
, : -, -, - -,

V22 4 . --, , ,
: , . 222 = 8, V32 8 .
V42 2 4 V52 2 5 . , 1-5

V12 V22 V32 V42 V52 = 21 2 2 23 2 4 25 = 2 4 8 16 32 = 62 .



116

: IV
n k-

Vkn n k .
5.1.3. ,
. () .

V340 40 3 = 64 000.
: 40 , 10 :
(), (), ()

().
2, 3, 4, 5, 6, 7, , ,
.
, ,
.

40 .
,
404040 = 64 000.
k n ( k n) ,
, . , ( )
n k

Vkn n (n 1) ... (n k 1) .
k ,
. , k .
5.1.4. n = 26 , k = 3: , .
.
, . 26
= 262524 = 15 600?
: , . 26
. . 25 .
. 24 . 262524
.


117

: IV
5.1.5. 100 1000 ?
?
: : 0, 1, 2, ..., 9.
. , ,
1-9. , ,
. ,
. 998 = 648 100 1000
.
648 , 1, 3, 5, 7 9 .
.
. . ,
, .
, . 588 = 320 . ,
100 1000 320 .
5.1.6. 41. a, b, c, d, e.
: V24 4 3 12 . 12
b 12 c. , xyz zyx,
: 1. abc, 2. abd, , 12. aed, 13. bac, , 24. bed, 25. cab, , 36. ced. : 37. dab, 38.
dac, 39. dae, 40. dba, 41. dbc. dbc.
5.1.7. S = {a1, a2, , an}, k N, 1 k n.
k- S k-
. Vkn

n!
, n! n (n 1) ... 2 1 .
(n k )!

k- S k- .
Vkn n k .
20 5.1.9.
1. = {1, 2, 3, 4, 5}. ()
.
2. ?
: 32 , 8 . (),
(), () ().

20

, : , IV
, 4. , 1994, . 167-177.


118

: IV
VII (7), VIII (8), IX
(9), X (10), , , .
, .
3. , 12 .
?
4. 0,
1, 2, 3, 4? ?
5. 0 10 000 ? ,
?
6. A = {a, b, c, d, e, f}. abc
A, fed .
i.
ii.

45. ( ) A?
88. A?

7. = {a1, a2, a3, a4, a5, a6}.


? 21. ?
5.2.
.

1 2 3 4
. ,
4 1 3

, A {1,2,3,4} f
2

2413.
.
( ) ( ) .
, n n . Vnn ,
P(n) , Pn , n! . (-) -,
.

n! = Vnn = n (n 1) ... 2 1 .
, .
, n,
n

n
n! 2n .
e
n = 8 1,04%, .

119

: IV
5.2.1. ?
: 52 , , : 2, 3, 4, 5,
6, 7, 8, 9, 10, J (), Q (), K () A (), : ( -),
(-), ( -) ( -).
.
. n = 52

52
n! 2 52

2,71828

52

= 8 10 67 .

, n k

Vkn

n!
.
(n k )!

,
, . 5! 5 4 3 2 1 = 120 ( ).
, A = {a1, a2, , an} n k,
1 k n.
A n k ( k - ) k
A. k = n
A.
5.2.2. A = {a1, a2, , an} n . Vkn
k , Pn , C kn
k n (1 k n). :
a) Vkn n(n 1)...(n k 2)(n k 1) ,
b)

Pn n! n (n 1) ... 2 1 ,

: a) () k S n
. , n ,
n 1 ( ), n 2 ( ),
, k- n (k 1) = n k + 1 .
. b) a) k = n.


120

: IV
21 5.2.3. 1, 2, 3, ..., 2n
?
: (, , ...) n! .
n! n
(, , ...). , n! n! = (n!)2.
5.2.4. n ,
?
: a b. ()
. , a b, .
(n 1)!, ()
n 1 . , a b
2(n 1)!. n! 2(n 1)! = (n 2)[(n 1)!].
5.2.5. 8 ,
?
: 8 . i1
(. ), i2 ,
..., i8 . i1i2...i8
1, 2, ..., 8 ( i1, i2, ..., i8 ,
). , i1i2...i8
1, 2, ..., 8,
.
85617342.
8! = 40 320.
k x1x2xk (xi A, i = 1, 2, , k)
. A,
a1 k1 , a2 k2 , ..., an kn ( k1 + k2 + + kn = k),
( n ) k.
k > n
. , .
5.2.6. A = {a1, a2, , an}.
a) Vkn k (k > 1),

Vkn n k .

21

: , , 1998.


121

: IV
n
b) Pk1 ,k2 ...,kn A, a1

() k1 , a2 k2 , .... an kn ,

Pkn1 ,k2 ,...,kn

(k1 k 2 ... k n )!
.
k1!k 2 !...k n !

:
a) ( ) x1x2xk n ,
. (
). , n
n
...
n nk .

b) ,
k1 + k2 + + kn , (k1 + k2 + + kn)!. ,
(i = 1, 2, , n) ai,
ki!. ,
k1!k2!...kn!
A.
k1 + k2 + + kn .

k1!k 2 !...k n ! Pkn1 ,k2 ,...,kn (k1 k 2 ... k n )! .


, V23 3 2 = 9 P23, 2,1

(2 2 1)!
5 4 3 2 1
5!
=
=
= 30,
2!2!1!
2 2 1
4

.
5.2.7. 4 , 3
2 ?
.
: P49,3, 2

9!
1260 .
4!3!2!

5.2.7. 0, 1, ..., 6?
: , V37 7 3 = 343
0, V27 7 2 = 49. ,
V37 V27 = 343 49 = 294.
5.2.8. () ?

122

: IV
: ( 28.09.2009.)

( )
X00 - X - 000.
A, E, J, K, M, O, T.

. 7,
10 . .
710107101010 = 4 900 000.
5.2.9. 11 ?
: , 10! .
5.2.10. ?
: . P24, 2

4!
6 . ,
2!2!

, , , , , .
5.2.11. 2
, 5 ?
:

7!
35.
3!4!

5.2.12.
1. 5. .
2. 5. 3 .
3. 2, 3
4?
4. A = {1, 2, 3}
2 ?
5. 0, 1
2?
6. 483. aaabcdeee.
5.3.
A n k A k .


123

: IV
5.3.1. A = {a1, a2, , an} n . Vkn
( ) k , C kn k n
(1 k n).

C kn

Vkn
n(n 1)...(n k 1)
=
.
k!
k!

: x1x2xk k k!
{x1, x2, , xk}. , C kn k! Vkn , .
5.3.2. 39/7 ?
: 39/7 39 , 7 . ,
(
), 7. 39 . C739

39 38 ... 33
=
7!

15 380 937.
5.3.3. 9 1, 2, ..., 9.
3 . ? 52.
3. 9 .
: 3. 9 C39
1 C 28

V39 9 8 7
=
84 .
3!
1 2 3

87
28 . 2 C 27 21
1 2

. , 1 2 28 + 21 = 49 . 50. (345), 51.


(346) 52. (347). , 52. 347.
5.3.4. 7 4 6 ,
. ?
: k = 2, 3 4 , C k4 = 6, 4 1 .
6 k = 4, 3 2 , C 67 k = 35, 35 21 . ,
4

C
k 2

4
k

C 67k = 6 35 4 35 1 21 = 371.

5.3.5.
1.
.


124

: IV
2. (Four of a Kind), .
.
: 13 22: (), (), () ().
3. 20 10 ?
4. 20 30
10 15 ?
5. 8 7 5 ,
. ?
6. .
:
a. ;
b. ;
c. .
7. n . ?
8. n .
9. n . ,
.
a. ?
b. ?
10. k m x n, k m, k n ?
5.4.
5.4.1.

n(n 1)...(n k 1)
,
k!

n
,
k

n, k N, 1 k n . n k, . . 1 .
0
k

C kn .
k

.
5.4.2. :
22

: http://www.elemenat.com/cyr/wiki/index.php?title=__


125

: IV

n!

a)
,
k k!(n k )!

,
b)
k
nk

n n n 1

.
c)
k k 1 k 1

: a) , 1 0! = 1, =
0
n



n n(n 1)...(n k 1)
n(n 1)...(n n 1)
= 1. 1 k n , =
=
n!
k!
k

n(n k )...(n k 1) (n k )!
n!
=
, .

k!
(n k )! (k!(n k )!
n

n!

n!

n!

=
b) ,

=
.
k k!(n k )! n k (n k )![n (n k )]! k!(n k )!
.

n n

n!

n!

=
c) k! (k 1)!k
=

k k 1 k!(n k )! (k 1)!(n k 1)!

n!
n 1
(n 1)!
n!
1
1
=
=

=
k!(n k 1)! n k k 1 k!(n k 1)! (k 1)(n k ) (k 1)!(n k )!
n 1
(n 1)!
. .
=
(k 1)![(n 1) (k 1)]! k 1
5.4.3. c) .
: , Ckn Ckn11 Ckn1 , k = 1, 2, , n. A = {a1, a2,
, an}. n k
, . C kn . . , k
, 1. 1 C kn11 ( {a2, , an} n 1
k 1, a1), 1 C kn 1 .
.

n n 1

, n = 1, 2, 3, k = 1, ..., n:
5.4.4.
k k k 1
a) ;

126

: IV
b) .

Vn

n (n 1)(n 2)...(n k 1) n (n 1)((n 1) 1)...((n 1 (k 1) 1)


=
=
(k 1)!
k
(k 1)!

: ) k
k k! k

n Vkn11
n n 1
. b) : Ckn k n Ckn11 .

=
k (k 1)! k k 1
k n ,
. k n, . C kn ,
k. n , n
1 k 1 , n C kn11 .

n(n 1) n 2

.


k k (k 1) k 2
5.4.5. n ?
: 2n. : , 1 n, .
- 2, 22...2, n 2,
. 2n.
5.4.6. n :
n

k 0

k k n 2

n 1

: n = 0. n 1.
n

n n 1

n
k =

k 0 k
n

k 1

k k

n 1
= n 2 n1 .
j
j 0

n 1

n 1

k k k 1 = n k 1 = ( j = k 1) = n
k 1

k 1

5.4.7. n, k :
n

n 1

k 0

j k

k = k = k 1 .


127

: IV

j 1 j j

.
k 1 k k 1

: 5.4.2.(c)

j
=
j 0 k
n

j 1 j

1 0

k 1 k 1 = k 1 k 1 +
j 0

2 1 3 2


+
+ ... +

k 1 k 1 k 1 k 1

n 1 n n 1 0 n 1

=

.
=

k 1 k 1 k 1 k 1 k 1

n 1
(k+1)
k 1

n+1. j

= k, k+1, , n . k+1 . ?
k

k 1
. ? ... ,
k

k+2

n+1 . ? .
k

5.4.8. n, k :
n

j 0

j k

Vk j Vk j

1
Vkn11 .
k 1

: k!.
5.4.9. n, r :

n 1 n n 1
n r n r 1
...
=
=
.
i 0 i
0 1
r r

n i

= ( j = n + i) =

i 0 n
n r 1
.
= ( 5.4.2.b) =
r

n i

i 0 i
r

n r 1

n 1

nr

j n

n = ( 5.4.7.) =

, (A. . Vandermonde, 1735-1796).


5.4.10. m, n, r :


128

: IV

m n r m n


r j 0 j r

.
j

m n
r
r

: m n

. . , j = 0, , r
, r j .
.
5.4.11. n :
2

n
2n
.

j 0 j
n
n

n
: =
j 0 j
n

n n

2n

j n j = n .
j 0

n k- , ,

n k 1
.
k

C kn
5.4.12.

A = {1, 2, 3, 4, 5}
C 35 ( )

B = {1, 2, 3, 4}
C 34 ( )

5!

5 4 3 2 1

4 3 1 6
=
3 3

C 35 =
=
= 10 C34
3
(
3
3
!

(
2
5

1
)

(
3
2

)!
1
)

6!
= 20, .
3!(6 3)!


129

: IV
.
.

,
c) (5.4.2). ,
,
b) . , a)

:

, , , ,
10. , 1665. .
...
,
2
2
2
1665.
(a b) a 2ab b
,
(a b) 3 a 3 3a 2 b 3ab 2 b 3
. ,

(a b) 4 a 4 4a 3b 6a 2 b 2 4ab 3 b 4
...

n
n
(a b) n a n k b k .
k 0 k

, ,
- 10. :
n = 0, (a + b)0 = 1(ab)0,
, a b.
n = 1, (a + b)1 = a + b.
n,
n + 1.

(a b) n1 a(a b) n b(a b) n ,

[(a b) n1 ] jk [(a b) n ] j 1,k [(a b) n ] j ,k 1 ,


[ f (a, b)] jk a j b k f (a, b) . j k n

. j k n 1 , ( j 1) k n j (k 1) n ,
k


130

: IV

n n

n 1


. , j k n 1 ,

k k 1 k
( j 1) k n j (k 1) n , 0 + 0 = 0.
n 1 n 1

n1k k
a
(a b) n 1
b ,
k 0 k

C kn k A n
k

.
5.4.13. 23 A
n 2n.

: A k = 0, 1, 2, , n , , .
k

P(A)

k 0

k . a = b = 1,

n
n
n 1 n1 n n
1 1 1 =
(1 1) n = 1n 1n1 11 ...
0
1
n 1
0
n
n
2 n , .

k 0 k
n n

k 0

k . ,

5.4.14. ... (1) n 0 .


0
1
n
: a = 1 b = -1, (1 1)
n

(1)
k 0

k 0

k 1

nk

(1) k ,

n
0 .
k

5.4.15. 13. ( 2 3 3 )15 .

23

5.4.5.


131

: IV

15
T13 2
12

2 3

1512
2

12
3

15

k
15 k
15
2
2
3 3 , k = 12. ,
k
k 0

15

15
= 2 2 81 = 73710 2 .
12

5.4.16.
1.

(n 1)!
n(n 1) .
(n 1)!

2. {5, 6, 7, 8}?
3. :
a) 256.
b) 153.
4. (2n)!! n!2 n , (2n)!! 2 4 ... (2n) ,
.
5. :

2n
2n
,
n (n 1)
n
n 1
n n n n 2
2

,
b)
k 1 k k 1 k 1
n n 1 n 2 n 3


,
c)
n n n n 1
a)

n n 1

n m

n m 1
.
m

...

d)
0 1
m

n 1 n 1 n 1
:
:
5 : 5 : 3 .
m 1 m m 1

6. m n

n n

7. 8 .
3
2
1
[7]
8. 1 3x .
4


132

: IV
9. 11. ( 2 3 3 )15 .
10. x8 (3 x)12 .
[40095]
15

11. x 3 x .
x

15 4 11
2 3 ]
4

[
16

x
.
12.
3 x

[ ]
13.

x
3

x 1

.
15

[ ]
8

1
14. x
.
x

[ ]
16

x .
x

15.

[3294720x-4]

16. 2 x x 3 x .

[ 1120x

22
3

x 1

17. 2 x 2 2 135,

22.
[n = 6, x = -1 = 2]


133

: IV
18. (1 x) n :

a) 2 3 ... n ,
1
2
3
n



n n n
n


0 1 2
n

b)

... .
1
2
3
n 1


134

: IV

6.
X XI
, .
(1623-1622, (1601-1655)
(1629-1695). (1654-1705)
.
XVIII (1667-1754),
(17429-1827), (1777-1855).
. (1702-1763), . (1707-1788), (1781-1840)
.
XIX
. ,
(1821-1894), (1856-1922) (1858-1918).
( ),
. ,
XX .
. . 1933.
6.1.
.
, ,
.
, .
(. trial) ,
, .
(. event) , ,
. , , .
, , .
()
(sample space) , .
6.1.1. .
(. head)
(tail), S1 = {, }.

S2 = {(, ), (, ), (, ), (, )}.
S2 .
6.1.2. T V% .
(T,V). :

135

: IV
a. a T b, 0 V 100; b. T> 0, , 0 V 100; c. (a T -5 V 75%) (0 T b) .

Q
, Q ( ) . , ,
.
A A A
. A A .
A .
AB ( AB) A B
A, B.
AB ( A + B)
. , . , A
B AB = . A A , A A
.
A\B A,
B.
6.1.2. 1 6.
, 1 , 2 , ..., 6
. Q = {1, 2, 3, 4, 5, 6}. :
= {2, 4, 6}, B = {1, 3, 5}, 4 C = {5,

6}. A B Q , B C {1 , 3 , 5 , 6 } , B C {5 } , A {1 , 3 , 5 } B .
6.1.3. . :

A B A B , A B A B .
: A B A B , ,
A B , . A B .

136

: IV
A B . .
.
, .
A B

AB

A \ B

A \ B AB .
, .

A B . A B,
A , B, B ,
A, A B. A B B A, A B
A = B.
6.1.5.
1. , :


137

: IV
1. ,
2. ,
3. .
2. . 4 6,
4 6. .
3. .
. .
4. a, b, c, d. . A
: , B : b .
1.
2.
3.
4.

.
A + B.
AB.
A\B.

5. , B
. A B.
6. A, B, C E :
1.
2.
3.
4.
5.

A + A = A,
A + E = E,
A + = A,
A + B = B + A,
(A + B) + C = A + (B + C).

7. .
. B . :

A , A A , A A , AB , A B , AB , AB , A \ B , A \ B , A \ B , A \ B .
8. A, B, C E :
1.
2.
3.
4.
5.

AA A ,
AE A ,
A ,
AB BA ,
( AB)C A( BC ) ,

6.

A( B C ) AB AC .

9. A, B, C . :


138

: IV
1. A, B, C ;
2. A, B, C ;
3. A, B, C .
6.2.
, ,
.
.
n0 , n(A)
. 0 n(A) n0. ,

n( A)
,
n0

p0 ( A) . .
24 6.2.1. 30 , , , , , , , , , , , , , , , ,
, , , , , , , , , , , , , . 19 , 11.
n = 1000 , , 494 ( 506 ). n = 10000
, 5039 .
1/2.
.
,
F. , p(.)
p(A) , .

.
, F p( A) 0 ,

n( A)
0 . , ()
n0

, n() n0 , p() 1 . A B ,

n( A B) n( A) n( B) ,

n( A B) n( A) n( B)
a n0

n0
n0
n0

. p( A B) p( A) p( B) .
25 6.2.2. p(.) F
, :

24
25

http://www.elemenat.com/cyr/wiki/index.php?title=__, 1 0.
Zoran A. Ivkovi: MATEMATIKA STATISTIKA, III izdanje, Nauna knjiga, Beograd, 1976.


139

: IV
i.

: A F, p( A) 0 ;

ii.

: p() 1 ;

iii.

: A1, A2, (AiAj = , i j),

p Ak P( Ak ) .
k
k
.
, .
.
6.2.3.
1.

p() 0 A A p( A) p( A ) = p( A) p() .

2.

p( A) 1 p( A) A A p( A) p( A) p() 1 .

3. A B p( A) p( B) . , A B B A AB ,
, p( B) p( A) p( AB) . p( AB) 0
p( B) p( A) .
4. F p( A) 1. A 3.
5.

p( A B) p( A) p( B) p( AB) . A B A AB B AB AB ,
. p( A B) p( A) p( AB)

p( B) p( AB) p( AB) . p( AB) ,


.

26 .
6.2.4.

p( A B C ) = p( A) p( B) p(C) p( AB) p( AC ) p( BC ) p( ABC ) .


: A B C ( A B) C , ( A B)C AC BC ACBC ABC p( A B C )

= p( A B) p(C ) p(( A B)C) = p( A) p( B) p( AB) + p(C ) p( AC BC ) =


p( A) p( B) p(C) p( AB) p( AC ) p( BC ) p( ABC ) .
26

A B A B . ,

A B AB .

140

: IV
{1 , 2 ,..., n }
, . F F = P() = {, 1 ,..., n , 1 2 ,..., 1 2 ... n }
2 n (. 5.4.4.).
, . p(1 ) = p( 2 ) = ... = p( n ) ,

1
k = 1, 2, ..., n. ,
n
A k1 k2 ...km .

1 2 ... n p( k )

. p( A) p( k1 ) p( k2 ) ... p( km )

m
. ,
n

. ,
.
6.2.5. , 1 10 .
. 10
?
: (,)

10
45 . A {(1,9), (2,8), (3,7), (4,6)}
2

.
4, p( A)

4
.
45

6.2.6. . .
(-), (+).
.

I
+
+
+
+
+
+

II
+
+
+
+
+
+

III
+
+
+
+
+
+

IV
+
+
+
+
+
+
-

V
+
+
+
+
+
+
-

:

() (),
. ,

5
10 .
2
:
+ + + + + +.
,

2
.
10

6.2.7. , . 100 ,
5 ( A), 65

141

: IV
( B), 30 ( C).
?
: p(A + B) = p(A) + p(B) =

= 0,70.

6.2.8. 52 4 .
.
: 4 . A
4 . 52 4

52
n() 270 725 ,
4
48
n( A) 194 580 , . 48 4.
4
194 580
, p( A) 1 p( A) = 1
= 0,28126... .
270 725
.
6.2.9. AB a, C D.
C D A.
: AC = x, D = y. A-D-C-B,
A-C-D-B, ,
CD CA , | y x | x .
y x , x y x , . y 0 .
y x , y x x , . y 2 x .
,
. , I Oxy.
a. ,
P() a 2 ,

a
2
2 a ,
P( S )
2
4
2
2
a
3a

P( S ) a 2
. ,
4
4
a

p( S )

3a 2 / 4
P( S )
=
= 0,75.
P()
a2


142

: IV
6.2.10. 1 2
20 . ?
:
. 60 . je |x y| 20.
x>y
y>x
x y 20
y x 20
y x 20
y x + 20
(y = x 20)
(y = x + 20)
.
P() 60 2 3600 ,

40 40
1600 , P( A) 3600 P( A) 2000 .
2
P( A) 2000
, p( A)

0,56 .
P() 3600
P( A) 2

6.2.11.
1. 1, 2, 3, 4, 5 . :
A. 3 .
B. 1, 2.
2. 52 . :
A.
B.
C.
D.

.
.
.
.

3. 90 , 12 .
?
4. 110 7 ,
?
5. 12 7 . .
:
A. ;
B. .
6. 12 7 .
?


143

: IV
7. . 0,65 0,70.
?
8. . 0,7 0,5
0,4. ?

9. p

p( Ak ) , k = 1, 2, 3, ( ).
k

10. . :
A. ;
B. ;
C. B.

18 90
: ; 4.
2 2

: 1. 24/120, 12/120; 2. 1/52, 26/52, 13/52, 13/52 + 13/52; 3.

12 7

2
2
7
110
; 5. ,
( 7 103) /
19
2
2

2

12 7

1 1 ; 6.
19

2

12 7
2
7
; 7. 0,895; 8. 0,91; 9.
24

7

A1 A1 A2 A1 A 2 A3 ... A1 ... A j 1 A j A j .

6.3.
:
a.
b.
c.
d.

,
,
,
.

a.
, A B.
p(B|A) B ,
.
6.3.1. 120 30 .
, , . :
A. ;
B. .


144

: IV
B ,
. , :

p( B | A)

29
30
= 0,24... , p( B | A)
= 0,25... .
119
119

, ,
nA, nB, nAB A, B, AB , n .
B :

n AB
nA

n AB
n .
nA
n

p( B | A)

p( AB )
, p( A) 0 .
p( A)

p( AB) p( A) p( B | A) , p( BA) p( B) p( A | B) .
AB BA () .
b.
A = {1, 2, ..., n}
.
, 1, 2, ..., n

n

A
k 1

.
: 1, 2, ..., n

n

p( B) p( Ak ) p( B | Ak ) .
k 1

A B ,
k 1

p( B) p( Ak B) . p( Ak B) p( Ak ) p( B | Ak )
k 1

. .

145

: IV
6.3.2. . ,
, . , 2%
, 4 % .
, .
.
: . i. :
.
, t, ,
: 2n, n n . 4n . ,

2
2
2n , 2% =
n 4%
100
100
10n
4
=
= n/10 .
n ,
100
100
2% =

, ,
p() =

= 0,025.

ii. : . Ak k- (k = 1, 2, 3)
. A1, A2, A3 . p( A1 ) 2 p( A2 ) , p( A2 ) p( A3 )

1
1
1
, p( A2 ) p( A3 ) . ,
2
4
4
B : , p( B | A1 ) p( B | A2 ) 0,02 , . 2%,
p( A1 ) p( A2 ) p( A3 ) 1 . p( A1 )

p( B | A3 ) 0,04 .
p(B) = p( A1 ) p( B | A1 ) p( A2 ) p( B | A2 ) p( A3 ) p( B | A3 ) = 0,02 + 0,02 + 0,04 = 0,025.
c.
:
. . ,
p( A1 | B) ? ,
, .
(Bayes) : 1, 2, ..., n

p( A j | B)

p( A j ) p( B | A j )
n

p( A ) p( B | A )
k 1

, j = 1, 2, , n.


146

: IV
p( A j B) p( B) p( A j | B) p( A j ) p( B | A j )

p ( A j | B)

p( A j ) p( B | A j )
p ( B)

, p(B) ,

.
6.3.3. (6.3.2.) .
.
?
: p( A1 | B) =

p( A1 ) p( B | A1 )
0,5 0,02
=
= 0,4.
0,025
p( B)

6.3.4. :
1.
2.
3.
4.

3 , 2 , 2 , 3 ;
1 , 3 , 5 , 2 ;
2 , 4 , 1 , 2 ;
4 , 1 , 6 , 3 .

. :
A. ;
B. .
: : .
Kj : j- (j = 1, , 4). p( K1 ) ... p( K 4 )
+

. p(A) = +

= 2

+ +

= , p(A|K2) = 2

1
, p(A|K1) =
4

, p(A|K3) = 2 = , p(A|K4) = 2

= 0,1198.

, : , B :

1 1

p( K1 ) p( A | K1 )
p( K1 | A)
= 4 5 = 0,4174.
0,1198
p( A)
d.
, B

p( B | A) p( B) .
B


147

: IV

p ( A | B) =

p( AB ) p( A) p( B | A) p( A) p( B)
=
=
= p(A) ,
p( B)
p( B)
p( B)

B. , A B

p( AB) p( A) p( B) .
6.3.5. . : B :
.
6.3.6. 32 . :
A. ;
B. .
A B ?
: , . p( A)

p( AB )

8 1
4 1
, p( B)

32 4
32 8

1
. , p( AB) p( A) p( B) , A B .
32

, n N A1, A2, , An Ak1,


Ak2, , Akm p( Ak1 , Ak 2 ,..., Akm ) p( Ak1 ) p( Ak 2 )... p( Akm ) .
6.3.7. , , ,
. .
:
A. ;
B. ;
C. .
A, B C ?

2 1
1
, p( AB ) p( BC ) p( AC ) ,
4 2
4
1
p( AB) p( A) p( B) , ., A, B C , p( ABC )
4
1
p( A) p( B) p(C ) .
8

: p( A) p( B) p(C )

, .


148

: IV
27 6.3.8.
1. . 3?
2. 5 , 4 6 . 3 ,
?
3. . :
A.
B.
C.
D.

;
;
;
.

A C, A D, B C, B D, .
4. 3 . ,
: 0,8, 0,7 0,6.
?
5. 6% , 70%
. ?
6. 60% 40% .
0,3 0,4.
.
7. 30% , 25% .
1% , 1,2%, 2%.
?
8. . 1 6 ,
. 3 , 2 1 , 4 2
.
i.
ii.

.
, .

5 15

: 1. : {3,6}, B : {2,4,6}, p(A|B) = (1/6)/(3/6) = 1/3. 2. p( A | B) /


. 3. A C
3 3 91
p(C) = , p(C|A) = 1/2; A B p(A) = ,, p(A|D) = ; B C
p(B) = , p(B|C) = 2/3; B D p(B) = , p(B|D) = 1. 4. P(ABC) = p(A)p(B)p(C) = 0,80,70,6 =
0,336. 5. : , B : ; p(A) = 0,94, p(B|A) = 0,70, p(AB) = p(A)p(B|A) = 0,940,70 =
0,658. 6. A, B : , . S : . P(A) =
27

Ivana. Kovaevi: Verovatnoa i statistika sa zbirkom zadataka, Beograd, 2011.


149

: IV
0,6, p(B) = 0,4, p(S|A) = 0,3, p(S|B) = 0,4 p(S) = p(A)p(S|A) + p(B)p(S|B) = 0,60,3 + 0,40,4 = 0,34. 7. S :
, A, B, C , , . P(A) = 0,30, p(B) = 0,25, p(C) =
0,45, p(S|A) = 0,01, p(S|B) = 0,012, p(S|C) = 0,02, p(S) = p(A)p(S|A) + p(B)p(S|B) + p(C)p(S|C) =
0,015. 8. , B , . P(A) = 2/6, p(B) = 4/6, p(X|A) = 2/6, p(X|B) = 4/6, p(X) =
p(A)p(X|A) + p(B)p(X|B) = 1/5; p(A|X) = p(A)p(X|A)/p(A) = 1/5.
6.3.9.
1. . .
6?
2. m n . ,
.
,
i.
ii.

,
.

, .
3. p(.|A), p(A) > 0 F, . 6.2.2.
4. p( A1 , A2 ,..., An ) p( A1 ) p( A2 | A1 ) p( A3 | A1 A2 )... p( An | A1 A2 ... An1 ) .
5. p(B) 6.3.1.
6. I x y , II u v .
I II, II
. ?
7.
11% , 9% , 2%
. , :
A. ;
B. ?
A B ?
8. A B , A B , A B .
9. 14 . , ,
. :
A. ,
B. ,

150

: IV
C. .
9. A B . A

0,4 0,5 . B
0,1, 0,2 0,3 . A
90% , B 70%
.
?
10. I 0,1, II
0,05. . :
a. ;
b. ;
c. , .
11. : , , . :
,

. 0,3, 0,2, 0,1.

, .
12. ,
. .
.
13. , .
. ,
.
14. 3 . m1 n1 , m2 n2
, .
. : 1. , 2. .
15. . 35%, 40%,
25% . 90% , 95%,
85%. :
A. ,
B. , .
16.
.
p1, p2 p3.
?


151

: IV
17. 1 0,5.
2 1 : 1 ,
2 0,7, 1 , 2 0,8.
.
18. , ,
p. .
.


152

: IV

7.

. , . X : R.
() ()
(). ,
, .
7.1
,
.
7.1.1. X X()
. X, Y, Z, .
7.1.2. , X .
= {, , , }, 0, 1 2.
() = 0, () = () = 1 () = 2.
S = [-1,5; 1,5]. {XS} = {-1,5 X 1,5} = { : -1,5 X
1,5} = {, , }, p{XS} = p{-1,5 X 1,5} = p {, , } = .

. , .
x1, x2, , xn, p1, p2, , pn,
p1 + p2 + + pn = 1, .

x
X : 1
p1

p2

...
.
...

1
1
2

2
.
1
2

x2

, (6.4.2.)

0
X :
1
4

28 7.1.3. (. )
. (+), ().
, , .
. .

28

Zoran Ivkovi: Matematika statistika, Nauna knjiga, Beograd, 1976.


153

: IV
:
.
: = {+, +, +, +, }. (+) =
1, ( +) = 2, ( +) = 3, ... , X : RX = {1, 2, 3, }. {X =
n} n 1 , n- .
, p{X = n} = p(n) = = ( )

31
, () p(n)
44

3 1
p(n)
4 n 1 4
n 1

n 1

, n .

n 1

1 .

1
X :
3
2
4

3
43

3
44

...
.
...

A = {2, 4, 6, }

3 1
p( A) p(2k ) =
4 k 1 4
k 1

2 k 1

3
1
1
1

.
= 3 =
1 5
16
k 1 16
1
16

29 7.1.4. x 2 y 2 1 Oxy.
. .
: ,
r .
, (,) r

x2 y 2 , 0 r 1.

,
.

29

M. Obradovi i D. Georgijevi: Matematika (za IV razred srednje kole), Zavod za udbenike, Beograd 1994.


154

: IV
, I (r1 , r2 ) , 0 r1 r2 1 ,
X I
r1 x 2 y 2 r2 , .
P( X I ) r2 r1 ,
. ,

, .
2

(r2 r1 )
2

p( X I )

= rr 2 r1 2 .

, .
, ,
(, ) I.
, . p{ | X () x} 0 , x
I. .

F ( x) p( X x) , < x < ,
x X.
,
. :
1.

0 F ( x) 1 ;

2.

F () 0 , F () 1 ;

x1 x2 F ( x1 ) F ( x2 ) ;
4. p(a X b) F (b) F (a) ;
3.
5.

lim F ( x) F ( x0 ) .

x x0

7.1.5.
R (. 7.1.4.). () .
:
. 0 x R, {|X()=x} x
x2.

p( X ( ) x)

x 2
x2
=
, 0 x R .
R 2
R2



155

: IV

0, x 0
x 2
F ( x) 2 , 0 x R
R
1 x R.

f(x)

f ( x)dx 1 F ( x) f (t )dt .
b

p(a X b)

f ( x)dx , f(x)
a

dF ( x)
F ( x) f ( x) , . F(x) ( ) f(x).
dx
7.1.6. - < x < ,

f ( x)

1
.
(1 x 2 )

a. f(x).
b. p(-1<X<1).
:
. .
b. p(1 X 1) =
1

1
dx
1
1
=
arctg
x
| .
1 (1 x 2 )
2
1

, F(x)
f(x),
, .
(a,b) f(x)
(a,b)
.
30 7.1.7.

30

Svetozar Vukadinovi: Elementi teorije verovatnoe i matematike statistike, Beograd, 1978.


156

: IV
1. .
, 0,5
.
2. , 3/4 , .
.
.
3. () . (
). .
4. 3 5 .
1 2,
1 - ,
2 - .

ax 2 , x [0,2]
5. f ( x)

0, x [0,2]
. , p(0 X 1) .
6. (a,b) (a b
a < b),

0,
1
f ( x)
,
ba
0,

xa
a x b.
x b.

F(x) f(x) F(x).


7. ( )

F ( x) 1 e

xm
x0

, x 0.

:
a. f(x).
b. xp F(xp) = p.
p.
c. x .
.

157

: IV
8. ( ) :

F ( x) 1 e

x2
2 2

.
a. f(x).
b. x0,5 F(0,5) = 0,5.
( 0,5).
c. x ( ).
:

0 1 2 3 4

F ( x)
1. X :
1
4
6
4
1

16 16 16 16 16

2.

xi

0,

x0

1
,
16
5
,
16
11
,
16
15
,
16

0 x 1
1 x 2
2 x3

3 x4
x4

1,
1

pi

3. ,
. ,
,
2

1
1 1 1
1 1 1
p( X 1) , p( X 2) 1 , p( X 3) 1 , ...
2
2 2 8
2 2 4
, p( X x)

1
, x = 1, 2, 3, . :
2x


158

: IV

x 1

x 1

p ( X x) = 2

4. F1 ( x)

0,
20
,
56
50
,
56
1,

0 x 1

F2

1 x 2

x2

1
2

0,

x0

25
,
64
55
,
64

0 x 1

x0

1 .

1
1
2

1 x 2
x2

1,
2

5. ,

ax

dx 1 , a

3
. , f(x)
8

, = 2,
.

F ( x)

p(0 X 1) =

8x
0

0,
x a
,
6. F ( x)
ba
1,

0,

x0

x3
,
8

0 x2

1,

x 2.

1
dx = F (1) F (0) .
8

xa
a x b .
xb

, .


159

: IV

xm

m m1 x0
m 1
m
7. a. f ( x)
x0 .
x e , x 0. b. x p [ x0 ln(1 p)] m . c.
x0
m

8. a. f ( x)

x2
2 2

. b.

2 log 2
1,18 . c. .
log e


.
, (T)
(V). ,
. T() V() , , . (T, V).
7.1.8. X = X() Y = Y()
. (X,Y)
.
Oxy.
() () .
:
Y\X
y1
y2

yj

x1
p(x1,y1)
p(x1,y2)

p(x1,yj)

p
i

ij

x2
p(x2,y1)
p(x2,y2)

p(x2,yj)

xi
p(xi,y1)
p(xi,y2)

p(xi,yj)

1.

7.1.9. 4 () , 3 2 ,
. X, Y .
2- (X, Y).


160

: IV
: p(x = 0, y = 0) = p{ } = ( ) /( ) = 3/36, p(0,1) = p{ } = ( ) ( )/( ) =
32/36, p(0,2) = p{ } = ( ) /( ) = 1/36, p(1,0) = ( ) ( )/( ) = 12/36, p(1,1) = ( ) ( )/( )=
8/36, p(2,0) = ( ) /( ) = 6/36. (1,2), (2,1) (2,2) . .

10/36 ( ),
14/36 (
).

X\Y
0
1
2

0
3/36
12/36
6/36

1
6/36
8/36
0

2
1/36
0
0

(, ) .
7.2
31,
(Bernoulli) ():
i.
ii.
iii.
iv.

n = 1, 2, 3, ;
( );
p q = 1 p ;
.

, p(A) = p
. A ,
p q = 1 p. n = 1, 2, 3, , (

n) A , 2n.
Sn = 0, 1, 2, , n. Sn
.
, :
(Poisson) . ,
.
7.2.1. 3 :
. n = 3, p(A) = , S3 (0, 1 2).
2 3 8 A A A , A AA , AA A , A A A , AAA , A AA , AA A , AAA . ,

A k (k = 0, 1, 2, 3), A 3 k
k

p q

3 k

. ,

3
p k p{S 3 k} p k q 3k (k = 0, 1, 2, 3).
k
31

http://www.gimbl.com/lat/binomnaras.php


161

: IV
{S 3 0} {S 3 1} {S 3 2} {S 3 3}

3
3
3
3
p0 p1 p2 = q 3 pq 2 p 2 q p 3 1 .
0
1
2
3
Sn. ,
n p, B(n;p).
B (3; ).
, {Sn = k}, - n
k ( n k ).
Sn, . pk = p{Sn = k}, k = 0, 1, , n.
{Sn = k}, ,
p(A(1) (2) A(n)) = p(A)p( )p(A) = pqp = pkqn-k,
q = p( ) = 1 p, a A(j) - , = 1, 2, ..., n.
k- (n-k)- ( )
pkqn-k, :
pk = p{Sn = k} = ( ) pkqn-k, k = 0, 1, , n.
=

( )

= 1.

32

n
Pn, x , p p{S n x} p x q n x , q = 1 p.
x
n

n, x, p

x 0

Pn, x 1, p
Pn, x , p

1 . ,

n! p x 1q n x 1
n! p x q n x n x p
:

.
( x 1)!(n x 1)! x!(n x)! x 1 q

:

32

nx p
1 , . x np q , Pn, x 1, p > Pn, x , p .
x 1 q

Svetozar Vukadinovi: Elementi teorije verovatnoe i matematike statistike, Beograd, 1978.


162

: IV

nx p
1 , . x np q , Pn, x 1, p = Pn, x , p .
x 1 q

nx p
1 , . x np q , Pn, x 1, p < Pn, x , p .
x 1 q

, Pn , x , p x np q ,
x np q , x . np q
. ,
:

np q np q 1 np p .
7.2.3. Sn, :
a. n=39, p = 0,935;
b. n = 15, p = .
: a. np q 39 0,935 0,065 36 np p 39 0,935 0,935 37 ,
36 37. b. np q

5
8
, np p .
3
3

7.2.4. 1% .
0,95?
n
: Pn, x1, p 1 Pn,0, p , Pn,0, p q , Pn, x1, p 0,95

1 q n 0,95 , q n 0,05 . n

log 0,05
= 298.
log 0,99

7.2.5. 6% . 5
, ?
: 3 ( p = 0,06)

P5;3;0,06 . Pn, x 1, p

nx p
Pn, x , p = 0, 1 2, :
x 1 q

5 0,06
P5;0;0,06 (1 0,06) 5 0,734 , P5;1;0,06
0,734 0,234 ,
1 0,94
3
4
P5; 2;0,06 0,0638 0,234 0,030 , P5;3;0,06 0,0638 0,030 0,002 .
3
2


163

: IV
, n > 5,
Pn , x , p . , , n > 50
.
7.2.6. 0,01. 100
. :
a. 5 ;
b. 10.
: : p(A) = p = 0,01 :
p( ) = q = 0,99. 100 (
), . :
a.

100
0,015 0,9995 ;
p{S100 5} =
5

b. 10 {S100 0} {S100 1} ... {S100 10}

{S100 k} =

k 0

10

10

pk =
k 0

100
0,01k 0,99100k .
k
k 0

10


n . . n > 50 np < 15
, ( n np 10) . n ,
.
7.2.7. np = > 0,

n
k
p k p k q nk e
, k = 0, 1, 2, , n .
k!
k
P()
. .
, , , ,
.
X[a,b] [a,b].
{0,1,2,}
33:
a. : X[a,b] [a,b],
b a;
33

Zoran Ivkovi: Matematika statistika, Nauna knjiga, Beograd, 1976.


164

: IV
b. : [a1,b1] [a2,b2] , X[a1,b1] X[a2,b2]
;
c. : lim

t 0

p{ X [t , t t ] 1}
0 ,
t

t .
, X[0,t] = X(t) P(t),
> 0 .
7.2.8. X(t) t ,
20t. 6
15 ?
: X(t) P(20t) t = 0,25 X(0,25) P(5)
. p{X(0,25) = 5} = e

56
= 0,1462.
6!

7.2.9.

i.

S np

p n
x

npq

ii.

S np

p a n
b
npq

1
2 npq
1
2

x2
2

x2
2

, n ;

dx , n .

7.2.10. (7.2.2.)
, = np = 1000,01 = 1 < 10, :

1
0,003 ;
5!

a.

p{S100 5} e 1

b.

100

0,01k 0,99100k

k 0 k
10

10

e
k 0

1
= 1,000.
k!

p( X x)

k 0

k!


165

: IV


166

: IV
7.2.11. 14 400 7 428 .
np = 14400 = 7200.
: S14400 n = 14400 .
= {S14400 7200 7428 7200},

S
7200

240 ,
p A p{228 S14400 7200 14400} = p 3,8 14400
60

1 1
npq 14400 = 60. , a = 3,8
2 2

b = p A

1
2

x2
2

dx .

3, 8

, (Laplace)

( z )

1
2

x2
2

dx , (z ) = 1 ( z ) =

1
2

x2
2

dx .

( ), z
+, z = 3,8 0,00007235. , pA = 0,00007.


167

: IV


168

: IV

7.2.12.
1. p = 0,01 20 ,
10 20 ?
[0,9910 0,904]
2. .
Sn .
3. 3 , ?
[1-( ) =

0,7]

4. 0,59.
.
[0,2]
5. 30 , 6 , 4 .
4 . :
a. ;
b. .
6. p{X = k} = 2-k, k = 1, 2, .
a. p{X };
b. p{X 4};
c. p{X 5}.

169

: IV
7. , .
0,6 0,7. ( )
.
8.
0,5 0,5 .
.
.
9. . 5%
, 3%. .
12 3 .
10. .
, :
a. 3 3 ;
b. 2 5 .
11. .
4 , .
0,85. a , a/3.
, b .
c , .
12. 3 , 15
. 15
P(0,8).
.
13. 0,03.
120 . :
a. ;
b. 3;
c. , 0,9
120 ?
14. 1200 . 1/365.
6 .
15. 150 , 0,8.
,
100 , .


170

: IV
7.3.
,
, .
7.3.1.

x
X 1
p1

x2

...

p2

...

xn
,
p n

p
k 1

1,

M ( X ) xk pk .
k 1

, f(x),

- < x < ,

M (X )

xf ( x)dx .

,
. () ,
, .
7.3.2. . ,
. () .
: 1, 2, 3, ..., k, , ,
,

, ...,

, . () =

2
k 1

1 k
.
2 k 1 2 k 1

1
, |x| < 1,
1 x
1
1
1
, , x = , () =
= 2.
1 2 x 3x 2 ...
2
2 1 2
(1 x)
1
2

1 x x 2 x 3 ...

7.3.3.
.
:


171

: IV

1
,

f ( x) b a
0,

x ( a, b)
x ( a, b)

xdx
b2 a2
ab
() =
=
=
.
ba
2(b a)
2
a
b

7.3.4. F(x) X,

F (x p ) p
p. 0,5 X, .
:

F (M e ) F ( x0,5 ) p( X M e ) 0,5 .
, 0,25 0,75, . .
7.3.5. , .
,
.
, , .
, M(X) = Me, .
x a b, y = ax + b
. x xk, y yk = axk + b,
, . p(yk) = p(xk).
7.3.6. M (ax b) aM ( x) b ,
: . p(ax + b) = ap(x) + b,

M (ax b) =

(ax

b) p( xk ) = a xk p( xk ) b p( xk ) .
k

x
k

p ( x k ) M ( x) ,

p( x

) 1 M (ax b) aM ( x) b ,

.
7.3.7. X Y


172

: IV

M ( XY ) M ( X )M (Y ) .
: XY xkyj pkpj ,
(pkj = pkpj) M ( XY )

x
k

x j p kj =

p k y j p j = M ( X )M (Y ) .

.
7.3.8.
, .

M (| X |) | M ( X ) | .
: 1, 2, ..., xn, p1, p2, ,
pn, |X| |1|, |2|, ..., |xn|, .

|xk|pk xkpk |xk|pk, k = 1, 2, , n,


M (| X |)

k 1

k 1

| xk | pk M ( X ) xk pk

| x
k 1

| p k M (| X |) ,

.
7.3.9. (a,b),
.
: f(x) (a,b) . ,
b

af ( x)dx xf ( x)dx bf ( x)dx

a M ( X ) xf ( x)dx b
a

f ( x)dx 1 .
a

1 , 2 , x1 M(X) x2.
. x = c, .
f(c + x) = f(c x), M(X) = c.
7.3.10. () = np


173

: IV

n
p( X k ) p k (1 p) n k , k = 0, 1, 2, , n.
k
: () =

n
x p x (1 p) n x =

x 0 x
n

x 1

(n 1)! p (1 p)
( x 1)!(n x)!
x 1
n

np

x
x 1

n! p x (1 p) n x
=
x!(n x)!

n! p x (1 p) n x
=

x 1 ( x 1)!( n x)!
n

n x

. x 1 = y () = np

(n 1)! p y (1 p) n 1 y
=

y!(n 1 y )!
y 0
n 1

np[ p (1 p)]n1 = np .
7.3.11. M (X )

p( X x) e

xe

: M (X ) =

x 0

x
x!

, x = 0, 1, 2, .

x 1
e x

=
= e
= e e .
x!
x 1 ( x 1)!
x 1 ( x 1)!

, .

xk
e x .

k
!
k 1

7.3.12. M (X ) ,

f ( x)

: M ( X )

te

t2

dt

t2
2

xe

1 x

1 x

dx = [

, - < x < .

t]=

(t )e

t2
2

dt =

dt . , . M (X ) .

7.3.13. . 1, 2, ..., n,
xk < k = 1, 2, , m xk > k = m+1, m+2, , n,

p( X )

M (X )

: xk k = 1, 2, , n ,
n

M ( X ) xk pk >
k 1

k k pk >

k m 1

p k =

k m 1

k m 1


174

: IV

n

p( X )

k m 1

pk 1
k 1

k m 1

M (X )

, p( x ) 1

M ( x)

.
7.3.14.
1. :
i.
ii.

< a, M(X) < a;


> a, M(X) > a.
[ xkpk < apk]

2. ,
.
[ : 0 1,
p q = 1 p. M(X) = ]
3. , 20 ?
[70]
4.

xe x ,
f ( x)
0,

x0
x 0.

2 x
[ x e dx 2 ]
0

5. ,

e x ,
f ( x)
0,

x0
x 0.
[ M (X )

, Me

ln 2

6.

175

: IV

f ( x)

1
, - < x < .
(1 x 2 )

, .
7.

2 cos 2 x,
f ( x)

0,

x (0, )
4

x (0, ).
4

.
[ 2 cos 2x (0,/4), .
p( X M e ) 0,5 , p(0 X M e ) 0,5 ,
Me

. 2 cos 2 xdx = sin 2M e 0,5 , M e


0

12

8.

3 x e 3
, = 0, 1, 2, ... ;
x!

i.

p ( X x)

ii.

12
p( X x)
3 3

iii.

3 5 4
p( X x)
x 9 9

iv.

x 4

5 3 x

p( X x)
, = 0, 1, 2, 3.
9

3

x 1

, = 1, 2, 3, ... ;
x

3 x

, = 0, 1, 2, 3;

[3; 3; ; ]
9.
i.

f ( x) 2 x , 0 < x < 1;

ii.

f ( x)

iii.

, 0 < x < 1;

2
f ( x) 6 x(1 x) , 0 < x < 1;


176

: IV

iv.

f ( x)

1
, 1 < x < .
x2
[ , , , ]

7.4
,
.
7.4.1. .

D( X ) M ( X M ( X )) 2
() .
()

D(X ) .
7.4.2.

0
X :
2

1
1
3

2
.
4

9
2

: M ( X ) 0
2

11
2
1
4 11

1 2 , D( X ) M ( X M ( X )) 2 = M X =
9
3
9 9
9

1
450 50
11 2 11 1
11 4

2
2
2
=

0 1 2 = 3 (11 2 2 3 7 4) =
81 9 81
9 9
9 3
9 9 9

(X )

5 2
.
9

7.4.3.

1
,

f ( x) b a

0,

x ( a, b)
x (a, b).


177

: IV
: (. 7.3.3)
2

(b a) 2
ab
dx
ab

. D(X ) = x
=
,

M (X )

2
b

a
12
2

a
b

(X )

|ba| 3
.
6

[ x

M ( X )]2 p k .

[ x M ( X )]

f ( x)dx . ,

.
7.4.4.

D( X ) M ( X 2 ) M ( X ) .
2

: D(X ) = M [ X M ( X )]2 = M {X 2 2M ( X ) X [M ( X )]2 } =

M ( X 2 ) 2M ( X )M ( X ) [M ( X )]2 = M ( X 2 ) [M ( X )]2 .
7.4.5. IA , :
IA = 1 , IA = 0 , p(A) = p.
p p p2 .

A
A.

1,
0,

: I A I A ( )

M ( I A ) = 1 p( A) 0 p( A) = p( A) p . , I A 2 I A
D( I A ) = M ( I A 2 ) [M ( I A )]2 = M ( I A ) p 2 = p p 2 .
7.4.6.

p( X x) e

x
x!

, x = 0, 1, 2, .

: M (X ) (. 7.3.11.) .
D(X ) = M ( X 2 ) 2 =

k
k 0

p k 2 =

k
k 0

1 k
1
e 2 = e k
k 1 2
k!
(k 1)!
k 1


178

: IV

= e

k 1

k 0

k!

(k 1)

k 1

k 0

k!

2 = e k

k 1

k 0

k!

k 0

k!

2 = ke

k 0

k!

2 =

M ( X ) e e 2 = 2 2 = .
7.4.7.

f ( x)

1 x

, - < x < .

: M (X ) (. 7.3.12.).
2

M ( X ) =

t2
2

x e
2

1 x

dx

dt 2 . M ( X 2 ) 2 2 . , D(X ) =

M ( X ) [M ( X )]2 = ( 2 2 ) 2 = 2 .
2

2
, .
, N(,2).
7.4.8. () :
i.

D( X ) 0 , D( X ) 0 X c ;

ii.
iv.

D(cX ) c 2 D( X ) , ;
D( X c) D( X ) ;
X Y D( X Y ) D( X ) D(Y ) ;

v.

: M ( X c) 2 D(X ) c M (X ) .

iii.

: iv. D( X Y ) = M ( X Y ) 2 [M ( X Y )]2 = M ( X 2 2 XY Y 2 ) [M ( X ) M (Y )]2 =

M ( X 2 ) 2M ( XY ) M (Y 2 ) [M ( X )]2 2M ( X )M (Y ) [M (Y )]2 =
M ( X 2 ) [M ( X )]2 M (Y 2 ) [M (Y )]2 = D( X ) D(Y ) .
v. M ( X c) 2 = M X M ( X ) M ( X ) c = M X M ( X ) M ( X ) c .
2

7.4.9. npq (p + q = 1):

n
p( X k ) p k (1 p) n k , k = 0, 1, 2, , n.
k


179

: IV
: M ( X ) np (. 7.3.10.).

n k nk
p q
= (
k 0
k
n
n 1 k 1 ( n1)( k 1)
p q
= np k
= ( j = k
k 1 k 1
n

I . M ( X 2 ) =

n 1

) =
k n
k
k 1

kn k 1 p
k 0

1, m = n - 1) = np

( j 1) j p

j 0

n 1

q nk

q m j = ( ) =

m
m
np j p j q m j p j q m j = (
j 0 j
j 0 j

m
m

m 1 j m j
m
p q
np m
p j q m j =
j 0 j
j 0 j 1

m
m 1
) =
j m
j
j

m 1 j 1 ( m1)( j 1) m m j m j
p q
np (n 1) p
p q = np(n 1) p( p q) m1 ( p q) m =
j 0 j 1
j 0 j

np(n 1) p 1 = n 2 p 2 np(1 p) . , D(X ) = M ( X 2 ) M ( X )2 =


np(1 p) n 2 p 2 (np) 2 = np(1 p) .
II . Ak k- , k =
1, 2, , n. Sn , n ,
(. 7.4.5.) Ik Ak Sn = I1 + I2 + + In.
D(Sn) = D(I1) + D(I2) + + D(In) ( 7.4.8. iv.). , D(Ik) =
p(Ak) p(Ak)2 = p p2 = p(1 p) = pq ( 7.4.5.), D(Sn) = npq, .

S n np
npq

Sn. , Z* Z,
N(, 2) N(0, 1) ,
. , .
7.4.10.
, > 0

p{| X | }

M (X 2 )


180

: IV

: f(x). M ( X 2 ) =

| x|

f ( x)dx

f ( x)dx = 2

| x|

f ( x)dx

f ( x)dx =

f ( x)dx +

f ( x)dx

| x|

| x|

2 p{| X | } .

| x|

X M(X)

p{| X M ( X ) | }

D( X )


. , (X ) =

D(X ) =

M X M (x) . ,
2

, , 34.

68% -
( ). 95%
2 . 99,7% ( 3, + 3).
,
() . .
X Y

X ,Y

M [( X M ( X ))(Y M (Y ))] M ( XY ) M ( X ) M (Y )
=
.
( X ) (Y )
( X ) (Y )

34

i.

| X ,Y | 1 ;

ii.

X Y , X ,Y 0 ;

http://sr.wikipedia.org/wiki/_


181

: IV
iii.

| X ,Y | 1 Y X ( < 0 X ,Y 1 > 0 X ,Y 1 ).

7.4.11. :
(0,6), (1,3), (2,1), (3,-2), (4,-3).
:


182

: IV
. A, B, C, D
E ,
-1 ( -1 1). ,
,
xi | 0 1 2 3 4 || 10
yi | 6 3 1 -2 -3 || 5
xi2 | 0 1 4 9 16 || 30
xiyi | 0 3 2 -6 -12 || -13
yi2 | 36 9 1 4 9 || 59

X ,Y

5 (13) 5 10

5 30 10 5 59 5
2

115

30 15

0,98976 .

7.4.11. (X,Y) () ABC


A(0,0), B(1,0) C(0,1). X Y.
: ABC P = , 1/ P = 2

2,

( x, y )

0,

( x, y ) .

(x,y) ABC, , . f ( x, y )

( )
x y = x (
I , y = x) yx.
. 2,
i- yi2x.
1

yi 2x x2dx x 2 | 1 .
i

, .
1

M ( X ) x 2 xdx
0

M (Y 2 ) y 2 2(1 y )dy
0

1
2
1
2
2
, M (Y ) y 2(1 y )dy , ( X ) x 2 xdx ,
2
3
3
0
0
1
1
1
, D( X ) M ( X 2 ) [ M ( X )]2
, D(Y ) M (Y 2 ) [ M (Y )]2
6
18
18

1
1 x

1
M ( XY ) M ( X ) M (Y )
3

x
dx

=
=
, X ,Y =
xyf
(
x
,
y
)
dxdy
2
xydy
dx

0 0

4
D( X ) D(Y )
0

, M ( XY )

1 2 1

4
3 3 1 .
=
1
2
18
35 7.4.12.

35

Zoran Ivkovi: Matematika statistika, Nauna knjiga, Beograd, 1976. . 51.


183

: IV
1. .
.
2. . , n .
. .
3. ( 1000 )

ex ,
f ( x)
0,

x0
x 0.

2 ,
5 < 0,5. ,
.
i.
ii.

.
.

4. . .
2 4 ..
. .
5. 5 10 ,
5 .. .
0,9 .
.
6. 8 .
.
. .
7.

p{ X k} a
i.
ii.
iii.

22
, k = 1, 2, 3, 4.
k!

.
.
5 .. n, :
1. n;
2. ?

8.

, 1 .

4 , 3 2 .


184

: IV
0 .
,

(1 x 2 ) ,
f ( x)

0,

x0
x 0.

5 .
9. .
() (5; 7). .
10. : 7.4.8. ii., iii.
11. p{| X M ( X ) | } , :

1
1
.
X :
1 / 3 2 / 3

1 1
,
X :
0,5 0,5

12. , () U(-1; 3).


185

: IV

8.

XIX , ,
, , , .
. ( ) (
), . ,
, .
, , ,
. 36 1998, ,
: , ,
37.
, ,
.
8.1.
" "
. , .
, .
( ) ,
.
, 38 ()
() . :
, .
, , (169 cm, 185 cm, 173 cm );
, . , ,
(, , , ...).
8.1.1. n , np nq (p + q = 1).
. ( p n).
: . ()
. , , ,
1 , 0 .
,
n p, np n(1 p) . ,

36

M. Lovri, J. Komi, S. Stevi: STATISTIKA ANALIZA (metodi i primjena), Banja Luka, 2006. str. 3
David Moore, George McCabe: INTRODUCTION TO THE PRACTICE OF STATISTICS, New York, 1998, str. Xviii.
38
( ), ( ).
37


186

: IV
m . m1

n .

, ,
, , . .
8.1.2. , 16. 2008.
:

.
11-20
9
21-31
16
25, 29, 45, 19, 36, 17, 60, 51, 39, 24, 15, 13, 31, 18, 24, 32,
37, 27, 23, 53, 41, 34, 29, 28, 52, 17, 55, 47, 34, 28, 22, 20,
31-40
12
64, 39, 38, 33, 24, 16, 27, 19, 26, 27, 25, 32, 26, 48, 54, 35.
41-50
4
51-60
6
61+
1
() , .
, .

. .
, .
,
.
360 48,
,
,
= (. ). :
67,5 120 90 30 45 7,5
-.


187

: IV
,

, .
, ,
20 40 .
, , .
:
,
;
( );
.
, , x1, x2, , xn ,
. , ,

= . ,

. , xk ,
fk,

,
, , ,
. ,

=
= 2,67857.

5
1
4
5
3
10
2
8
1
4
, . 13, 15, 16,
15. , . 13, 15, 16, 17,
,

= 15,5.

8.1.3. , 48
(8.1.2.) :
13, 15, 16, 17, 17, 18, 19, 19, 20, 22, 23, 24, 24, 24, 25, 25,
26, 26, 27, 27, 27, 28, 28, 29, 29, 31, 32, 32, 33, 34, 34, 35,
36, 37, 38, 39, 39, 41, 45, 47, 48, 51, 52, 53, 54, 55, 60, 64.
1558, , : 1558/48 32,4583. 48 , 24
, 9 , (24 27) . ,
, . 24 27, .
, 29 29 8 . ,
29, .
,
. - (. stem and leaf diagram).

188

: IV
8.1.4. 23, 45.6 7.89

23
45.6
7.89

2
45
7.8

3
6
9

8.1.5. 23
12 13 21 27 29 34 35 36 37 40 41 46 48 49 51 52 56 58 62 64 65 80 81
-

1 2
3
2 1
7
9
3 4
5
6
7
4 0
1
6
8
9
5 1
2
6
8
6 2
4
5
8 0
1
: 3|4 = 34
12. , 46. , 46.
8.1.6.
1. , ?
a.
b.
c.
d.

.
.
.
.

2. , ?
a.
b.
c.
d.

.
.
100 .
.

3. 12 450 . , 1200
. 35 .
a. ?
b. ?
c. .


189

: IV
4. . 120 ,
. 200 20
.
a. ?
b. .
5.
.
90
6 .
1200
.
a. ?
b. ?
c. ?
d. 20
?
6. ( ) 30 :
153
152
185

159
168
183

157
163
163

151
158
164

165
165
182

187
157
163

187
175
155

178
174
189

174
152
155

175
166
153

, 10 cm, .
7. ( ) 40 :
240
247
248
236

243
232
238
226

225
242
237
229

239
235
242
248

226
233
247
234

237
237
234
247

243
238
237
238

243
235
228
240

247
248
234
238

248
227
228
239

, 225-227, 228-230, 231-233, ..., .


8. 6 , 5
, 4 , 1 . .
9. , 6, 7 8.
10. -


190

: IV
616
780
810

679
944
449

330
738
381

653
770
465

508
338
648

260
914
393

525
783
823

157
502
821

518
162
972

672
706
102

981
996
100

748
210
465

11. :

, .
8.2.
, .
,
.
:
( = ),
(3),
(3).
,
. .

5
4
3
2
1

I
1
5
10
8
4

II
0
1
19
6
2

:
1. (Xmax Xmin) ;
2. (2
),
( );
3. (
),
;
4. (Q1, Q2, Q3) .


191

: IV
, .
M(X), D(X).
, I 5, 1, ,
4. je I
2

1 n
( xk ) 2 f k =

n k 1

1
75 2
75 2
75 2
75 2
75 2
(5 ) (4 ) 5 (3 ) 10 (2 ) 8 (1 ) 4 =
28
28
28
28
28
28

281
1,11508.
252
I I

1,05597.

II Xmax Xmin = 4 1 = 3. II =
2

1
75 2
75 2
75 2
75 2
(4 ) (3 ) 19 (2 ) 6 (1 ) 2 =
28
28
28
28
28

113
0,44841.
252
II 0,44841 0,66964. I II
.
( = 2,68), I II
(O , f , - ):
O
fII
(O )2fI
O
(O )2 fII
5,38
5
0
2,32
0
8,71
4
1
1,32
1,74
1,02
3
19
0,32
1,95
3,70
2
6
-0,68
2,77
11,29
1
2
-1,68
12,10
30,10
II:
12,10
2
2
I = 1,08
I = 1,04
II = 0,43
II = 0,66
(O ) (O) (
= 2,68) (I II) .
, (f) .
, . I II
n = 28.
O
5
4
3
2
1

fI
1
5
10
8
4

O
2,32
1,32
0,32
-0,68
-1,68
I:


192

: IV
, 2 =
, 2 =

= 1,075 1,08, =
= 0,432 0,43, =

1,04.
0,66.

, :

( xk ) 2

= f

xk

( xk ) 2

1
( f k xk ) , n .
n

,
, ,
. :

s n21

1 n
f k ( xk x) 2 sn1 sn21 ,
n 1 k 1

n . n n 1,
sn sn-1,

. ,

, , ,
2

,
(, ), .
25% (. ) Q1.
50% ,
. 75% (), Q3.
8.2.1. , :
1

10

11

12

13

14

15

16

17

18

19

20

21

22

23

24

25

26

27

28

1 1 1 1 2 2 2 2 2 2 2 2 3 3 3 3 3 3 3 3 3 3 4 4 4 4 4 5
1 1 2 2 2 2 2 2 3 3 3 3 3 3 3 3 3 3 3 3 3 3 3 3 3 3 3 4
Q1
Q2
Q3
28 , 7 , 14,
21. , 2
, . Q1 = 2, . , Q2 = 3
Q3 = 3. , .


193

: IV
39 8.2.2. (download) (
) 2006. 12
:
28,5 17,4 15,3 12,3 19,0 22,1 19,2 20,1 15,7 20,9 22,5 25,6.
, .
, .
12,3

15,3

15,7

17,4
Q1

Q2
Q3

19,0

19,2

20,1
Q2

20,9

22,1

22,5

25,6

28,5

Q3

15,7 17,4
19,2 20,1
19,65 , Q1
16,55 ,
2
2

22,2 22,5
22,30 .
2


, .
40 8.2.3. :

.
0-9
1
1
10-19
4
5
20-29
9
14
30-39
11
25
40-49
32
57
50-59
23
80
60-69
10
90
70-79
9
99
80-89
1
100
100 , 9 ,
10 19 , 20-29, ..., 80-89
.
, . 25 25
40.
,
. ()
, ()
. .

39
40

M. Lovri, J. Komi, S. Stevi: STATISTIKA ANALIZA (metodi i primjena), Banja Luka, 2006. str. 79
Fabio Cirrito: MATHEMATICAL STUDIES SL, International Baccalaureate, IBID Press, 1998, p. 549


194

: IV

, ( 100 )
( 25% ) .
25, 50 75.
, Q1, Q2 Q3, ,
40, 50 57. - IQR = Q3 Q1 = 57 40 = 17 .
, (. boxplot).
: (Minx), (Q1),
(Q2), (Q3) (Maxx).
41 8.2.4. 9, 8, 2, 3, 7, 6, 5, 4, 5, 4, 6, 8, 9, 5, 5, 5, 4, 6, 6, 8. : ,
, .
: , 2-3-4-4-4-5-5-5-5-5-6-6-6-7-8-8-8-9-9 (20 ).
10. 11. , . Q2 =

= 5,5.

41

M. Coad, G. Whiffen, J. Owen, R. Haese, S. Haese, M. Bruce: Mathematics for the international student,
Mathematical Studies SL, International Baccalaureate Diploma Programme, Haese & Harris Publications, Australia,
2004, p. 146


195

: IV

( 5. 6. ), . Q1 =
, . Q3 =

= 4,5.

= 7,5. - (interquartile range) = Q3 Q1 = 7,5 4,5 = 3.

: 2,
4,5, 5,5, 7,5, 9.
.
, , :

8.2.5.
1. , :

5
4
3
2
1
-

I
0
1
5
10
7
7

II
3
4
9
6
5
3

i.
ii.
iii.
iv.

I II
I2 II2

.

: () 7 (3) 30
.
[2,

; 0.85, 1.25; 1, 2; 2, 3; 3, 4]

2. (kg) A B :


196

: IV

A B , :
i.

ii.

() s n 1

f ( x x) ;
f 1
f (x ) .
()
f
2

iii.

[1.99, 2.00; 0.0552, 0.1877; 0.0547, 0.1858]


3. :
:
0-4
5-9
10-14
15-19
20-24
.:
2
7
10
13
12
i.
.
ii.
() .
iii.
.

25-29
6

[16.44; 6.74; 12, 17, 22]


4. .
i.
.
ii.
.
iii.
.
iv.
?
: .

1 9.99
10 19.99
20 29.99
30 39.99
40 49.99
50 59.99

3
11
25
36
31
14


197

: IV
5. - , :
0 1223
1 0112345
2 2336
3 15
4 7
: 2|3 23

50
50
60
60

136
015678
255
4

: 50|3 53

10 0 1 2 3
20 2 3 4 7 8 0
30 1 3 4
40 2 6
50 9
: 20|3 203

6. ( 0 10 ) 200 :

i.
ii.
iii.
iv.
v.

6 .
.
?
.
.

7. ( ) .
i.
ii.
iii.
iv.
v.
vi.
vii.

?
.
.
56 kg?
50-60 kg?

60 kg?
.

8.3.

(. 7.4.11.).
()


198

: IV
()

, .
,
,
.
.
. :
,
.
8.3.1. ,

y kx n


x ( )
y ( ) .

y 2 y1
, y y1 k ( x x1 )
x 2 x1

A(x1, y1) B(x2, y2) .

88 64
= 0.8,
190 160
y 64 0.8( x 160) , . y 0.8x 64 .
A(160, 64) B(190, 88), k

A B,
.
,
X Y, n (x1, y1), (x2, y2), , (xn, yn)

s xy
sx s y

s xy

( x x)( y y)
n

xy x y
n

- X Y,


199

: IV

( x x)

sx

( y y)

sy

y
n

x - X,

y - Y.

( x x)( y y)
( x x) ( y y )
2

, r


r2 = 0

0 < r2 0,25

0,25 < r2 0,50

0,50 < r2 0,75

0,75 < r2 0,90

0,90 < r2 < 1

r2 = 1
.

xy n x y
x nx y
2

ny



(r2).
.

8.3.2. (X )
(Y ). :
:
I
II
III
IV
X [g]:
1
3
5
6
Y [mm] :
2
3
6
6
.
: n = 4 ,

x 15 , y 17 , xy 77 , x

71 ,

85 .

xy, x2 , y2 :
:
x [g]:
y [mm] :
xy
x2
y2

I
1
2
2
1
4

II
3
3
9
9
9

III
5
6
30
25
36

15
17
77
71
85

IV
6
6
36
36
36

77 4
2

15 17

4 4

15
17
71 4 85 4
4
4

53
59 51

0,966. r2 0,934 .


200

: IV
8.3.3.
(X ) (Y) .
(2, 10), (3, 7), (7, 5), (4, 6) (9, 4). .
:
.
,
xy, x2 , y2, .
xy 136 ,
x 25 ,
y 32 ,

x
r

159 , y

136 5

226 .
25 32

5 5

32
159 5 5 2 226 5
5

= 12

x
2
3
7
4
9
25

y
10
7
5
6
4
32

xy
20
21
35
24
36
136

x2
4
9
49
16
81
159

y2
100
49
25
36
16
226

5
-0,894. r2 0,799
901


.
(8.3.1),
. , .
,
.
,
(. residuals). , y = y' y
' . ' ,
, y = kx + n.
() ()
y'' y'. , , . ()
, ().
, , B .
, x'1, x'2, , x'n y'1, y'2, , y'n, k
n y = kx + n. x'i yi = k x'i + n .
yi = y' yi , .
. (yi)2 .
k n (y1)2 + (y2)2 + ... + (yn)2 .
.
(. 7.4 : ).
8.3.4. Y X + , ,
[Y (X )]2 ,


201

: IV

(Y )
[ X M ( X )] M (Y ) .
(X )

Y X ,Y
:

f ( , ) M [Y (X )]2

(1)

f
f
0.
0,

(2)

(1) f(,) = M(Y2) + 2M(X2) + 2 - 2M(XY) 2M(Y) + 2M(X). , (2)

M ( X 2 ) M ( X ) M ( XY ),

M ( X ) M (Y ).

(3)

,
,

M (X 2 )

M (X )

M (X )

= M ( X 2 ) [M ( X )]2 = D( X ) 0 .

(3) .
(3) X +
Y X. Y X .

yy

s xy
sx

( x x) .

.
8.3.5.
A(1,4), B(2,5) C(4,7).
. .
x
y
xy
x2
y 16 ,
xy 42 ,
x 2 21 , n = 3.
x 7,
1
4
4
1
x
y
7 16 14
2
5
10
4
= 42
=
,
s xy
xy
n
3
3
4
7
28
16


202

: IV
7

16

42

21

sx x 2
2

= 21

72
14
x = 7 , y y = 16 .
=
, x
3
n
3
3
3
n

14
16
7

3 x , . y x 3 .
3 14
3
3

, x ()
y = kx + n. ( x1 xn)
, ,
, .
8.3.6. :
x
y
:

2
3

xy n x y
1

1
x 2
n

4
4

5
6

7
6

9
7

10
9

11
10

15
11

1
522 63 56
8
=
=
1
621 (63) 2
8

24
0,648649,
37
56 24 63 70
=
1,89189,

n y kx =
8 37 8
37
y kx n ,
y = 0,65x + 1,89.

8.3.7. X Y . ?
x
y
i.
ii.
iii.
iv.

1
2
3
4
6
7
8
9
1
3
2
5
6
5
7
8
(: 1-4) (: 6-9) .
.
, .
y x = 5?


203

: IV

xy n x y
x nx y
2

232 8
2

ny

40 38

8 8

19
260 8 5 2 213 8
4

47 2
0, 93766.
5 201

() r2 0,88 .
i. x1

1 2 3 4
23 25
10 11
, y1
, A1 , .
4
4
4 4
6789
x 2
,
4
6579
,
y2
4
32 27
A2 , .
4 4
y y1
( x x1 )
y y1 2
x2 x1
27 11

11
4 x 10 ,
y
4
32
10
4
4

4
4
8 x 41
y
, .

11 44

y = 0,73x + 0,93 ( ).
ii. y = 0,708x + 0,783 ( ).
iii. , x = 5 y = 0,708x + 0,783 = 4,323 ( ).
8.3.8.
1. X Y . ?
x
2
4
5
7
9
10
11
15
y
3
4
6
6
7
9
10
11
i.
(: 1-4) (: 6-9) .
ii.
.
iii.
, .
iv.
y x = 1?
[x = 63, y = 56, xy = 522, x2 = 621, y2 = 448, r = 0,9686, ]
2. () (). :
x
56
91
84
63
10
63
28
35
91
y
66
100
60
96
24
46
35
36
72

63
80


204

: IV

1
n

xy n x y s

i.

s xy

ii.

iii.
iv.
v.

r2 = 0,60. ?
.
.

s xy
sx s y

xy

= 493,78.

r =

= 0,7715.

3. (y) , 15 ,
(x C), :
x
16,7
16,1
15,6
15,0
14,4
12,8
11,7
11,1
10,0
y
24
21
19
18
19
15
15
12
11
i.
ii.
iii.

y y

s xy
s x2

( x x) ,

s xy
s x2

sy
sx

r , .

18,3.
.
[ = 56,6667, = 17,1111, sx = 4,0551, sy = 3,9845,
r = 0,9686, y = 0,9482x 36,6215]

4. ( mm)
( ), . ,
, :
(mm)
11
25
27
48
(t)
38
28
51
81
, :
= 180, = 270,
i.
ii.
iii.

= 1240,

= 2554,

= 9592.

.
.
.

5. . 5,
3 .
, (X,Y) :
Y\X
0
1
2
3
4
5

0
0
0,01
0,03
0,05
0,07
0,09
0,25
1
0,01
0,02
0,04
0,05
0,06
0,08
0,26
2
0,01
0,03
0,05
0,05
0,05
0,06
0,25
3
0,01
0,02
0,04
0,06
0,06
0,05
0,24
0,03
0,08
0,16
0,21
0,24
0,08
1

i.

: .

205

: IV
ii.
iii.

B: 3.
(. 8.3.4.).
[i. p(A) = p(X>Y) =
ii. p(B) = p(X<3) =

= 0,01 + 0,03 + 0,05 + + 0,05 = 0,75.


= 0,01 + 0,01 + 0,01 + + 0,04 =0,27]

6. 8.3.4. , , .
M[Y (X2 + Y2 + )]2 .
?
8.4.
, ,
: - .
.
, ,
.
,

.
- (2) 42 .
(ak) (bk) (k = 1, 2, , s) ,

(a k bk ) 2
.
bk
k 1
s

(ak) (bk). ,
, , . .
H0: , .
, , H1
.
, .
? 95%
() , = 0,05.
( = 0,05 ) ,
1 . :
1. ,

42

http://www.elemenat.com/cyr/wiki/index.php?title=-_


206

: IV
2.

( = 0,05 . = 0,10)
. , (. = 0,01),
.

(. contingency table) ( mn)


. 43
.
Xi i = 1, 2, , m ai1, ai2, , ain, Yj j = 1, 2, , n
a1j, a2j, , amj. =
=
,
, = =
= .
(A) 44:
A

Y1

Y2

Yn

X1

a11

a12

a1n

X2

a21

a22

a2n

Xm

am1

am2

amn

43
44

Karl Pearson: On the Theory of Contingency and Its Relation to Association and Normal Correlation, 1904.
. observed value table


207

: IV

bij

X Y
(B)
XY
i

45

Y1

Y2

Yn

X1

b11

b12

b1n

X2

b21

b22

b2n

Xm

bm1

bm2

bmn

, B
. ,
i= 1, 2, , m B

n
m

ail a kj
n
n
bij = l 1 k 1 =

j 1
j 1
XY

ail

a kj =
XY j 1 k 1
l 1

a
XY = a
XY
l 1

il

l 1

il

.
() (B)

2
ij

(aij bij ) 2
bij

(2) , 46:

(m 1)(n 1) .
, m n .
m 1 , m-
m 1. , n 1
. , 2- (m 1)(n 1) .
, 47 ( 5%)
2 48 ().
45

. expected value table


. degrees of freedom (df)
47
. significance levels
48
. http://www.medcalc.org/manual/chi-square-table.php
46


208

: IV

15

= 0.01
= 0.05
= 0.10

6.635
9.210
11.345
13.277
15.086
16.812
37.697
3.841
5.991
7.815
9.488
11.07
12.59
24.996
2.706
4.605
6.251
7.779
9.236
10.645
22.307
2
(),
() ().
H0: X Y ,
. , ,
, .
, < () ,
. 1%, 5% 10%.
, > () .
, .
8.4.1. 400 ,
. ( 22), :



32
188
4
176
36
364
H0: .

220
180
400

= 19.8,

= 200.2,

= 16.2,


19.8
16.2
36

= 163.8 B:


200.2
163.8
364

ij

(aij bij ) 2
bij

220
180
400

(32 19.8) 2 (188 200.2) 2 (4 16.2) 2 (176 163.8) 2

=
19.8
200.2
16.2
163.8
7.5171 + 0.7435 + 9.1877 + 0.9087 = 18.36.


209

: IV
= 18,36. = (2 1)(2 1) = 1. = 0.05,
2 (1) = 3.841. > (1),
, . ,
.
8.4.2.
. , , (, ) ,
: (7, 14), (9, 12), (13, 9), (14, 7). 5%
.
: : H0 . =
(4 1)(2 1) = 3. 5%, 0,05. H0 > 7,815.
:

1.
2.
3.
4.
B
1.
2.
3.
4.

7
9
13
14
43

10,6
10,6
11,1
10,6
43

14
12
9
7
42

10,4
10,4
10,9
10,4
42
21
21
22
21
85
21
21
22
21
85

, :
= 5,816
A
B
aij bij (aij bij)2
7,815.
7
9
13
14
14
12
9
7

10,6
10,6
11,1
10,4
10,4
10,4
10,9
10,4

-3,6
12,96
-1,6
2,56
1,9
3,61
3,4
11.56
3,6
12,96
1,6
2,56
-1,9
3,61
-3,4
11,56
:

1,223
0,242
0,325
1,091
1,246
0,246
0,331
1,112
5,816

, H0, .

(1 4).
1. 2.
,
.

8.4.3. 8.3.8.5. 2- (X,Y):


Y\X
0
1
2
3
4
0
0
0,01
0,03
0,05
0,07
1
0,01
0,02
0,04
0,05
0,06
2
0,01
0,03
0,05
0,05
0,05
3
0,01
0,02
0,04
0,06
0,06
0,03
0,08
0,16
0,21
0,24

H0: X Y .

5
0,09
0,08
0,06
0,05
0,28

0,25
0,26
0,25
0,24
1

: () . ,
(B) :
B
0
1

0
0,008
0,008

1
0,020
0,021

2
0,040
0,042

3
0,052
0,055

4
0,060
0,062

5
0,070
0,073

0,25
0,26


210

: IV
2
0,075
3
0,007
0,03

= 0,043.

0,020
0,019
0,08

0,040
0,038
0,16

0,052
0,050
0,21

0,060
0,058
0,24

0,070
0,067
0,28

0,25
0,24
1

= (6 1)(4 1) = 15, = 0,05


H0 > 24,996. , = 0, 043
. . , X Y
, 8.3.8.5. .
8.4.4. 2010. .
(Kendrick Meek), (Marco Rubio) (Charlie Crist) . 3069
.
12 ,
. :

140
188
183
76
587

338
494
510
178
1520

153
306
316
187
962

631
988
1009
441
3069

49, 12 (aij ) 12
(bij) 12
aij bij ij
----------------------------------------------------------140 120.689801238 3.08960469232
338 312.518735745 2.07761888742
153 197.791463017 10.1433860118
188 188.972303682 0.00500271432149
494 489.332029977 0.0445299771913
306 309.695666341 0.0441011973597
183 192.988921473 0.517016994697
510 499.732811991 0.210943022122
316 316.278266536 0.000244823225098
76 84.348973607 0.826392513272
178 218.416422287 7.47877459673

49

http://www.elemenat.com/lat/tabkontigen.php


211

: IV
187 138.234604106 17.2031008598
---------------------------------------------------------- 12 2 = 41.6407162903. = 6,
= 0.05 - (6) = 12.592. , 2 > (6),
.
8.4.5. .
1. 22 279

31
17
48


109
122
231

140
139
297

Pauling, L. (1971). The signicance of the evidence about ascorbic acid and the common cold.
Proc. National Academy of Sciences (U.S.A.), 68, 2678{2681.
Fienberg. S.E. (1980). The Analysis of Cross-Classied Categorical Data. 2nd Edition MIT
Press: Cambridge, MA. Reprinted by Springer-Verlag, New York (2007).

2. 33 .

18
28
14
60

20
51
28
99
12
25
9
46

50
104
51
205
Yule, G.U. (1900). On the association of attributes in statistics: with illustration from thematerial
of the childhood society, &c. Philosophical Transactions of the Royal Society, Series A, 194 257319.
3. 1973-74. .

Steinway

4
2
6

13
1
14

11
2
13

2
2
4

9
2
11

6
0
6

45
9
54
Fienberg. S.E. (1980). The Analysis of Cross-Classied Categorical Data. 2nd Edition MIT
Press: Cambridge, MA. Reprinted by Springer-Verlag, New York (2007).

212

: IV
4. .

0
1
2
3
4
0
78
144
204
211
179
1
106
153
126
80
32
2
130
92
55
15
3
125
38
7
4
104
26
5
50

593
453
392
306
211
Waite, H. (1915). Association of fingerprints. Biometrika, 10, 421-478.

5
45

45

861
497
292
170
130
50
2000

5. 222 .

:

156
107
84
31

84
133
156
209

240
240
240
240
Bartlett, M.S. (1935). Contingency table interactions. J. Roy. Statist. Soc. Suppl., 2, 248-252.
Bishop, Y. M. M., Fienberg, S. E., and Holland, P. W. (1975). Discrete Multivariate Analysis:
Theory and Practice. MIT Press, Cambridge, MA. Reprinted by Springer-Verlag, New York (2007).

6. 263 , 197778. 1980-81.

j1
j2
j3
j4

0
0
0
0
140
0
0
0
1
31
0
0
1
0
16
0
0
1
1
3
0
1
0
0
17
0
1
1
0
5
0
1
1
1
1
1
0
0
0
20
1
0
0
1
2
1
0
1
0
9
1
0
1
1
0
1
1
0
0
12
1
1
0
1
1
1
1
1
0
4
1
1
1
1
0
Haber, M. (1986). Testing for pairwise independence. Biometrics, 42, 429-435.


213

You might also like